NMAT 2022 Mock Test 4

You might also like

Download as pdf or txt
Download as pdf or txt
You are on page 1of 98

www.byjusexamprep.

com

Mock Test Solutions in English

Questions

1. Direction: Three statements are given below at A, B, and C . There may be some errors in the given
statements. Mark the correct statement as the answer.

A) There is still disparity among men’s and women’s salaries.

B) There are a lot of finer details still to be ironed out.

C) The spectators were shouting, and cheering, and waving flags of their favourite club.
A. Only A B. Only B
C. Only C D. Both A and B
E. A, B, and C
2. A) The builders should have provided clarity regarding the lack of raw materials for the building.

B) His bad behaviour was a once in a blue moon event.

C) A patient can refuse consent for a particular treatment at the any time.
A. Only A B. Only B
C. Only C D. Both A and B
E. A, B, and C
3. Direction: Given below is a group of sentences which are jumbled up. You are required to rearrange them
to come up with a meaningful paragraph.

A) The latest retail inflation numbers point to a slight softening in the pace of price gains, and that should
provide policymakers some solace that recent interventions appear to be working. Inflation measured by
the Consumer Price Index (CPI), slowed 75 basis points from April’s 95-month high of 7.79%, to 7.04% in
May.

B) While it is hard to quantify the effect that the RBI’s surprise 40 basis points interest rate increase of
early May had on prices, the Centre’s May 21 decision to cut the excise duty on petrol and diesel by ₹8
and ₹6, respectively, seems to have had an immediate impact.
www.byjusexamprep.com

C) Inflation in the transport and communication category of the CPI slowed by 137 basis points to 9.54%
last month.

D) This key category, with a weight of 8.59 that places it behind only cereals and housing, captures the
pump prices of the main transportation fuels, making it a crucial indicator of price pressures in the
economy.
A. ABCD B. ACDB
C. ABDC D. ACBD
E. ADBC
4. A) Two top American officials have sent a clear message that Washington views the ongoing India-China
border tensions as part of the broader geopolitical contest underway in the region.

B) described the level of Chinese activity in Eastern Ladakh as “eye-opening” and questioned its
intentions.

C) Then, at the Shangri-La Dialogue in Singapore, the region’s most high-profile security event U.S.
Secretary of Defence Lloyd J. Austin said Beijing was continuing to “harden its position” along the border.

D) On a visit to New Delhi, General Charles A. Flynn, Commander of the U.S. Army Pacific.

A. ABDC B. ACDB
C. ABCD D. ADBC
E. ACBD
5. Direction: Complete the Analogy

Request : Command
A. Propose : Stipulate B. Procure : Sustenance
C. Consign : Hand over D. Volunteer : Agree to take
E. Package : Acquire
6. Boundless : Limit
A. Curtailed : Length B. Ravenous : Appetite
C. Impeccable : Flaw D. Captivating : Interest
E. Kempt: Tempt
www.byjusexamprep.com

7. Direction: Three statements are given below at A, B, and C . There may be some errors in the given
statements. Mark the correct statement as the answer.

A) The stranger delivered the message instantly as if his life was dependent on it.

B) Their failure to reply to our letter seems to infer a lack of interest.

C) They both were widows, so there was no one to take care of them.
A. Only A B. Only B
C. Only C D. Both A and B
E. Only B and C
8. A) Numerous towns have isolated by the heavy blizzard.

B) None of what has been said should be took to indicate censure.

C) Rishabh has never been known for his humble nature


A. Only A B. Only B
C. Only C D. Both A and B
E. Both A and C
9. A) If you fill this form, you could take records out of the archive.

B) The young lovers have been trying to put some money beside for their marriage ceremony.

C) Many of the immigrant village is in remote zones.


A. Only A B. Only B
C. Only C D. All are incorrect.
E. Only A and C are correct.
10. Direction: Given below is a group of sentences which are jumbled up. You are required to rearrange them
to come up with a meaningful paragraph.

A) The resignation of Sri Lanka’s Finance Minister Basil Rajapaksa as Member of Parliament on Thursday
did not come as a surprise, given the adverse public mood he and the rest of his family, including his
brothers President Gotabaya Rajapaksa and former Prime Minister Mahinda Rajapaksa, have been
facing.
www.byjusexamprep.com

B) The violence was an outcome of the attack unleashed by supporters of the ruling Sri Lanka Podujana
Peramuna (SLPP) on peaceful anti-government protesters, who have been running the campaign “Go
home Gota [Gotabaya Rajapaksa]” over the country’s worst economic crisis.

C) It needs no reiteration that Basil Rajapaksa, regarded as the live wire of the SLPP, was perceived in
certain quarters as one of those responsible for the attack on the protesters.

D) But what was surprising was that he took a month to quit after unprecedented violence in Sri Lanka.

E) His political departure comes at a time when efforts are on to get Cabinet clearance for the proposed
21st Constitutional Amendment, which is aimed at empowering Parliament over the executive President,
apart from barring those holding dual citizenship from entering the legislature.
A. ABCDE B. ADBCE
C. BCDAE D. DACBE
E. EBCDA
11. A) The India Meteorological Department (IMD) has forecast a ‘normal’ monsoon for this year, or 99% of
the Long Period Average (LPA) of 87 cm.The IMD has a multi-stage monsoon forecast system.

B) The April forecast usually has little detail on how much rain is expected during each of the monsoon
months, and whether the rain will be lopsided or evenly distributed geographically.

C) The IMD usually shares this in late May or early June, just around the time the monsoon is imminent
over Kerala.

D) The forecast in April is, thus, only a general indicator and of little public utility.

E) A normal monsoon forecast this year is also predicated on the absence of an El Niño, a warming of the
Central Pacific linked to the drying up of monsoon rains.
A. ABDCE B. ABCDE
C. ACDBE D. ADBCE
E. AEBDC
12. Direction: In the given question, a part of the sentence is printed in bold. Below the sentence, alternatives
to the emboldened part are given as A, B, C, and D, which may help improve the sentence. Choose the
correct alternative out of the given options.
www.byjusexamprep.com

Choosing one option out of the given choices can be tricky, particularly when you want to do both things.
But it turns out your eyes can reveal your decision before you have even made it.
A. turned out to B. but it turned to
C. also it turning out D. No change required
E. but it turning out
13. The oil excavation operation corporation in the Middle East and Africa has prepared to act decisively to
help the government maintain law and order, given the recent history of the bloody feuds in the given
regions.
A. was prepared to B. has been preparing for
C. was prepared for D. has preparation for
E. has a preparation for
14. Direction: Complete the analogy:

Moult : Bird
A. Slough : Snake B. Overwinter : Tolerate
C. Mutate : Spider D. Shuck : Oyster
E. Word: Ply
15. Renounce : Pledge
A. Acquit : Quarry B. Defile : Memorial
C. Recriminate : Hero D. Rescind : Order
E. Defame : Acquit
16. Direction: Fill in the blanks with the most appropriate preposition from the given options:

The teacher said to me that you are looking at it all wrong. The first page _______ the book describes the
author’s profile and you have skipped the same.
A. at B. of
C. to D. from
E. over
17. Even though the accommodation package was way more expensive than what we originally thought, it
included admission _______ the golf course at no extra expense along with complimentary spa and
saloon facilities.
A. on B. in
www.byjusexamprep.com

C. to D. within
E. along
18. Now, the empire seems to be making an exception to pardon bin Salman of the act against Khashoggi,
though it has long been clear that the crown prince has collected all _______ absolute power in his hands,
chiefly over safety matters.
A. from B. with
C. to D. but
E. along
19. Direction: Read the passage given below and answer the questions that follow.

The Indian crypto asset industry has witnessed exponential growth over the last five years and today, the
country reportedly has 15 million crypto asset holders that have put in Rs. 6.6 billion in these crypto asset
holdings. India now has two crypto unicorns and over 350 crypto startups in what is clearly a flourishing
industry.

This report seeks to bring clarity around crypto assets as a technology and make recommendations for the
regulation of the Indian crypto asset market. Like all financial assets, crypto assets pose certain policy
risks that must be addressed through a coherent framework that balances the public interest with the need
to encourage innovation. Towards this end, the report recommends the induction of a safe harbour for
crypto assets, similar to the one inducted for internet intermediaries in the Information Technology Act,
2000 and the Copyright Act, 1957. The size of first-generation internet technology companies today is
testament to the enabling power of safe harbours as a tool to promote innovation. If the crypto asset
space presents the future of the internet, as many experts propound, it is imperative that the crypto
industry be afforded the same legal succour that allowed big technology companies to reach where they
are today.

In an effort to establish regulatory recommendations for India, the authors have carried out an exhaustive
study of international regulatory approaches and bucketed them into four categories. The first approach
involves accommodating crypto assets into existing frameworks as they stand. The problem with this
approach is that several crypto assets do not fall neatly into the definition of existing financial instruments
and therefore fall outside the regulatory ambit. The second approach is innovation-friendly steps taken by
some regulators to encourage startups to set up base within their jurisdictions and help the local crypto
industry grow. The third is crypto-specific legislation or amendments that address the concerns around
licensing and registration, definition, tax, money laundering and terror financing, and investor protection.
www.byjusexamprep.com

This approach is comprehensive and seems the most practical route and is being adopted by most
advanced financial jurisdictions.

The analysis of possible regulatory pathways under existing regulation in India reveals a similar problem.
It is likely that most prominent crypto assets such as Bitcoin would not be suitably accommodated as
securities, or commodities. While crypto assets may be recognised as capital assets, the frameworks that
currently regulate these assets in India would necessarily leave gaps in investor protection and service
provider registration.

The analysis of the way crypto markets work, reveals that the industry should be regulated through
exchanges. These entities account for the bulk of the volume of activity in most major crypto networks and
provide a point of entry for most retail and institutional investors into the crypto market. As such, they
should serve as the designates for regulation around the various policy risks presented by crypto assets.

Why does the author believe that the cryptocurrency industry is growing?
A. The next generation is likely to do more and more B. Cryptocurrency has provided its investors solid
transactions via cryptocurrency. returns.
C. The numbers on cryptocurrency use indicate that
D. Cryptocurrency is slowly turning into an asset.
there is huge scope for growth in the industry.
E. The numbers don’t lie as far as the crypto industry
is concerned.
20. The crypto industry needs to be provided legal support:
B. To reach the level of the big tech companies in
A. To reach the level of the equity market in India
India
C. To represent the future of the internet in the long D. To define a separate asset class that encourages
run more growth and investments
E. To get under the radar of big tech companies for
acquisition purposes
21. The primary purpose of the author of the passage is:
A. To talk about the need to legalise cryptocurrency B. To talk about the urgency to legalise
in India cryptocurrency in India
D. To talk about the growth prospects of
C. To talk about regulating cryptocurrency in India
cryptocurrency in India
www.byjusexamprep.com

E. To talk about the rise of the crypto industry in


India
22. It can be inferred from the passage that:
B. The regulatory framework on cryptocurrency is
A. The regulatory framework on cryptocurrency in expected to provide different cryptocurrencies a
India requires fine-tuning. certain distinction about the type of financial
instruments they represent.
D. The regulatory framework on cryptocurrency
C. The regulatory framework on cryptocurrency has
should not be at the expense of the equity and
been contentious throughout the world.
derivatives market.
E. The regulatory framework on cryptocurrency is
still in ascendancy.
23. Direction: Read the passage given below and answer the question that follows:

Since the end of the Cold War, once-frosty relations between the United States and India have blossomed
into a wide-ranging, multifaceted strategic partnership. Although the two countries are not formal treaty
allies, their diplomatic, defence, and developmental interests show signs of profound convergence. To
quote former U.S. Envoy to India Robert Blackwill and former President Barack Obama, respectively,
bilateral ties were transformed from being as “flat as a chapatti” to one of the “defining partnerships of the
twenty-first century.”

There were three principal drivers of this shift. First, the collapse of the communist model and India’s
embrace of globalisation and market economics in 1991 reoriented India’s focus westward. Second,
India’s emergence as a nuclear-armed power and America’s willingness to incorporate India into the
global civil nuclear regime, despite the fact it is not a signatory to the Non-Proliferation Treaty (NPT),
brought the countries closer together. Third, China’s rise as a regional power with global aspirations
convinced recalcitrant leaders in New Delhi and Washington, D.C. that they could either hang together or
they would ultimately hang separately.

Consequently, over a three-decade period, the United States and India overcame the hesitations of history
to develop one of the most wide-ranging bilateral partnerships in the world. The two countries currently
operate more than two-dozen joint working groups, traversing issues from higher education to space
exploration. As officials on both sides are fond of reminding us, the Indian military now conducts more joint
www.byjusexamprep.com

exercises with the United States than with any other partner. Total bilateral trade (in goods and services)
between the two countries grew from $20 billion in 2000 to $150 billion in 2019

The bilateral relationship has rested on four mutually reinforcing pillars. First, both countries espouse a
shared commitment to democracy and liberal values. Second, the two partners are mutually invested in a
rules-based global and regional security architecture, in stark contrast to the alternative advanced by their
common Chinese rival. Third, thanks to healthy two-way flows of capital, labour, goods, and services,
India and the United States have a significant stake in each other’s economic success. Fourth, the
relationship rests on robust people-to-people ties. The Indian diaspora may account for just 1 percent of
the American population, but it grew at a rate of 150 percent between 2000 and 2018

Unfortunately for the two nations, there is an accumulating body of evidence that suggests that at least
one of the aforementioned pillars—shared liberal democratic commitments—can no longer be taken for
granted. Recent anxieties about democratic backsliding across the globe have not left India untouched.
This is not to deny that American democracy too is at a nadir. The last 12 months have seen an outgoing
U.S. President refuse to accept the results of a democratic election, an insurrection targeting the seat of
the national legislature, and state-level measures to politicise elections administration. These threats to
American democracy are real and visceral. But America’s struggles are arguably of a lesser magnitude
than India’s given the relative strength of its institutions, elevated per capita income, and democratic
longevity.

Which of the following is not a principal driver for the blossoming of India-US ties?
A. India’s westward focus as it embraced
B. China’s rise and its global aspirations
globalisation and a market economy
D. China’s huge trade surpluses with both the USA
C. India’s emergence as a nuclear-armed power
and India
E. All of the above
24. According to the passage, which of the four mutually reinforcing pillars between India and the USA is most
at risk today?
B. Increased protectionism in trade as shown by
A. Entry for Indian professionals into the USA
both the countries
C. The weakening of liberal democracy in both the
D. Low migration levels of Indians to America
countries
www.byjusexamprep.com

E. India and the US on the cusp of political upheaval


25. According to the passage, America’s democracy backslide is _____.
A. more dangerous since it is a bigger power B. of a lesser magnitude than India’s
D. temporary as Donald Trump exited the White
C. imaginary and not real
House
E. far-fetched at best and hypothetical at worst
26. The author’s attitude towards the downslide of democracy in India is one of _________.
A. pessimism B. neutrality
C. Tacit support D. cynicism
E. jocularity
27. Direction: Read the passage given below and answer the question that follows:

National Education Policy (NEP) 2020 emphasises on the need for vocational training, both in schools and
universities. While CBSE has introduced a total of 38 vocational subjects at the senior secondary level,
guidelines for the Central Universities Entrance Test (CUET) reveal that the exam will test candidates on
only three related subjects. Educators fear that this may dilute the focus on vocational education and
reduce its value.

Neharika Vohra, vice-chancellor, Delhi Skill and Entrepreneurship University (DSEU), says, “Indeed,
CUET is not focussing enough on vocational subjects. A reason could be that not many higher educational
institutes (HEIs) offer courses mapped with vocational training given at schools. Still, at its starting point,
stakeholders are more concerned about figuring out finer details of CUET such as correlating marks with
courses, than focus on including vocational subjects.”

Kamlesh Singh, principal, Shanti Asiatic School, Kheda, Gujarat, says lack of faculty at HEIs may be
another reason. “If CUET will not test students on vocational subjects, students will not be interested in
opting for them. This will have a cyclic effect, as lesser number of students will lead to limited faculty at
both schools and HEIs. Gradually, NEP’s aim to remove focus from attaining only academic excellence
and making students job-ready will remain unfulfilled.”

Vohra says that this step by CUET will dissuade students from opting for vocational subjects. “Students
who opt for one or two vocational subjects based on interest will be at a disadvantage. Instead of being
tested for five subjects in CUET, they will only be tested for three or four, depending upon their choice of
www.byjusexamprep.com

subjects.” This, in turn, will discourage schools from offering vocational courses, because at the end, they
also have to showcase success stories, she adds.

Schools are still at the nascent stage of offering vocational courses. Anshu Mital, principal, MRG School,
New Delhi, offers many skill subjects to all classes from standard VI onwards. “We offer Marketing,
Photography, Tourism, Meal Planning, IT Application and more. However, CUET will test candidates on
only three vocational subjects, including Agriculture, Mass Media and Yoga. While Mass Media is still
being opted by various schools, the other two are not yet popular.”

Requirements for vocational courses vary depending upon the school’s geographical location, says Singh.
“We are based in a rural backdrop and currently offer only two vocational subjects, including Carpentry
and Computer Applications. Even in Carpentry, we are facing issues as only a handful have opted for it
while faculty speaks only in the local language, leading to hiccups in our English medium school.”

Source: https://timesofindia.indiatimes.com/home/education/news/why-cuet-must-focus-more-on-
vocational-subjects/articleshow/91435400.cms

What is the primary purpose of the author?


A. To focus on the need to provide schools with the B. To emphasise the need to focus on getting
means to pursue vocational subjects students interested in vocational subjects
C. To discuss the importance of vocational subjects D. To argue for greater emphasis on vocational
in achieving the goal of inclusive education subjects in the CUET
E. To persuade CUET organisers to pay attention to
minute details surrounding the vocational subjects
28. Which of the following is a possible reason behind CUET not focusing more on vocational subjects?
A. Schools do not have adequate resources to teach B. CUET administrators are still in the process of
these subjects. figuring out the details of CUET.
C. Students are not interested in the subjects D. The added costs of organising exams for a variety
included in CUET. of vocational subjects
E. CUET administrators are under pressure to
ensure the smooth conduct of the examination
29. What is the ultimate aim of the New Education Policy?
A. To make students rise above the shackles of B. To force students to choose from a variety of
academic excellence vocational subjects
www.byjusexamprep.com

C. To prepare the student for employment in the D. To focus on the needs of the poor and
industry underprivileged
E. To educate students about different career
prospects.
30. Which of the following can be inferred from the passage?
B. Ensuring that people pay attention to different
A. Implementing the study of various vocational
vocational subject offerings is a challenge for the
subjects is not easy as it has many aspects.
government.
C. Vocational subjects can provide students with the D. Requirements to teach vocational subjects are
much needed stimulus to excel in any job. quite huge.
E. All of the above
31. Direction: Fill in the blanks with the most appropriate answer option from the given options:

The award-winning statisticians built a model without any research but based on their ____, and it turned
out to be ____ correct as well.
A. realism, aesthetically B. intuition, intellectually
C. guesswork, scientifically D. experience, empirically
E. inundation, semantically
32. A human being is quite ___ creature, for the gloss of rationality that covers his or her fears and ___ is thin
and often easily breached.
A. a logical, delinquent B. a frail, insecurity
C. a valiant, dread D. an ambitious, morality
E. a sound, veneer
33. Historical fiction serves as a decent ___ among the avid readers, but when considering the opinion of the
majority, they feel it ___ from history and does not portray proper fiction.
A. text, clarity B. work, distorts
C. genre, deviates D. category, is grandiloquent
E. sort, weaker
34. Experts say that a ______ of hunting, habitat loss and food scarcity had led to the cheetah's
disappearance in India. The cheetah is the only large mammal to become ______ in the country since its
independence from British rule.
www.byjusexamprep.com

A. Permutation, glorified B. Combination, extinct


C. Likelihood, present D. Form, popular
E. Loss, endangered
35. Direction: Complete the analogy:

COWARD : CRAVEN
A. Fabricator : Flippant B. Naive : Gullible
C. Opponent : Caustic D. Judge : Impartial
E. Dope: Dapper
36. ENFRANCHISE : VOTE
A. promote : sell B. sterilise : kill
C. filter : decontaminate D. illuminate : see
E. enlighten : soul
37. Direction: Study the given information carefully, and answer the questions that follow.

The following pie chart shows the distribution of candidates (in percentage) who were enrolled for a
competitive exam in six different institutes: A, B, C, D, E, and F. The total number of candidates who had
enrolled in six institutes is 10,000 .

The following pie chart shows the percentage of candidates (out of those enrolled) who passed the exam,
for the six different institutes: A, B, C, D, E, and F.
www.byjusexamprep.com

In how many institutes is the number of candidates who passed the same?
A. 2 B. 3
C. 4 D. 5
E. None of the above
38. What is the approximate percentage of the total number of candidates who passed in the exams of the six
institutes?
A. 10% B. 12%
C. 14% D. 15%
E. 16%
39. What is the average number of female candidates who passed from institutes C, E, and F if the ratio of
male and female candidates in institutes C, E, and F are 2 : 3, 11 : 4, and 1 : 4, respectively?
A. 100 B. 109
C. 112 D. 115
E. 102
40. If the number of candidates who passed in the exam of another institute, Institute M, is 360 more than the
number of candidates who passed the exam in Institute F and if the candidates who enrolled in Institute M
was 25% more than the candidates who enrolled in Institute A, then find the success rate of candidates
who passed in the exam of Institute M?
www.byjusexamprep.com

A. 10% B. 15%
C. 20% D. 25%
E. 22.5%
41. John invested $P in a scheme called ‘Profit’ which offered 20% compound interest per annum for 2 years.
After two years, he invested the amount received from the scheme in another scheme that offered 10%
compound interest per annum for 1 year. If he received a total amount of $19800 at the end of the year,
find the value of 2P.
A. 20000 B. 22500
C. 25000 D. 15000
E. 30000
42. Find the number of real roots for the equation .
A. 0 B. 1
C. 2 D. 4
E. None of the above
43. Out of the 12 arithmetic questions and ‘x’ algebra questions, the probability of randomly selecting two
algebra questions is . Find the value of x.
A. 10 B. 11
C. 12 D. 13
E. 14
44. A cultural club consists of 60% singers, and the rest are dancers. A fund is raised for a cultural fest in the
club. 75% of the funds were raised by singers with an average of Rs. 1800 per person. What average fund
per person must be raised from the dancers to fulfil the demand? (Note: Each person mentioned is either
a singer or a dancer.)
A. Rs. 900 B. Rs. 1000
C. Rs. 1200 D. Rs. 1500
E. Rs. 1600
45. Simplify for x: = x.

A. B.

C. D.
www.byjusexamprep.com

E.
46. Direction: Study the given information to answer the questions that follow it.

The following table shows the branches of a multinational company (M) that has different branches
(offices) across five countries.

What is the total number of postgraduate employees in all the branches of the five countries?
A. 9500 B. 9575
C. 9600 D. 9675
E. 9765
47. In how many countries is the average number of employees per branch the same?
A. 2 B. 3
C. 4 D. 5
E. None of the countries
48. If the number of postgraduate male employees in China is 1800, then what is the percentage of female
employees in China who are postgraduates?
A. 70% B. 72%
C. 75% D. 78%
E. 74%
49. In which of the given countries is the percentage of women employees to the total number of employees
(both male and female) in that country the second highest among the five countries?
A. USA B. India
C. Russia D. China
E. None of the above
50. John and Jessica completed a certain work together. Out of the $940 received by both of them, Jessica
www.byjusexamprep.com

received $376. If they completed the work in 12 days, find the number of days required by Jessica alone
to complete the work. (Assume that John and Jessica are paid according to the amount of work done by
them.)
A. 20 days B. 24 days
C. 25 days D. 30 days
E. Cannot be determined
51. The HCF of two numbers is 18. If the LCM of these two numbers is 1386, which of the following can be
the sum of these numbers?
A. 324 B. 584
C. 784 D. 1404
E. More than one of the above
52. A decor shop sells cloth that can be used as curtains or bedsheets. 50% of the available units of cloth in
the shop can be used as curtains, whereas 70% of the available units of cloth in the shop can be used as
bedsheets. If 120 units of cloth can be used as both, then find the total units of cloth available in the shop.
A. 480 B. 500
C. 600 D. 800
E. Cannot be determined
53. Nicky calculates her profit percentage on the selling price, whereas Kathy calculates it on the cost price.
The difference between the profits earned by both of them is Rs. 400, and the selling prices of both of
them are the same. If both of them get 20% profit, what is the selling price?
A. Rs. 12000 B. Rs. 10000
C. Rs. 9000 D. Rs. 8000
E. Rs. 9500
54. If the roots of the equation x3 + bx2 + cx + d = 0 are in AP with the common difference of 12, and the value
of ‘d’ is –1215, then find the value of ‘c’.
A. 530 B. 531
C. 532 D. 533
E. 534
55. Working for 12 hours a day, a group of men can complete a piece of work in 18 days. If 10 persons are
excluded from the group, the remaining group can complete the same work in 24 days, working 15 hours
www.byjusexamprep.com

a day. If 5 men are added to the original group, find the number of days required to complete the same
work if they work for 10 hours in a day.
A. 24 days B. 15 days
C. 20 days D. 16 days
E. 18 days
56. John has a seven-seater SUV. He, along with his six friends, wants to go on a road trip in that SUV. If only
four of them know how to drive an SUV, find the total number of ways in which the friends can sit in that
SUV. (Assume that the SUV will only be driven by the people who know how to drive it.)
A. 2400 B. 2700
C. 2620 D. 2880
E. 2440
57. Find the value of mn if , and m and n are positive integers greater than 1
A. 42 B. 45
C. 48 D. 51
E. 52
58. Ankit gave a discount of 25% on each article and was able to gain a profit of 33.33%. However, a
customer bargained and was able to get 16 articles on a purchase of 12 articles. Find the actual profit/loss
percentage.
A. 12.5% loss B. 8.33% profit
C. 12.5% profit D. 15% loss
E. No profit / No loss
59. Vijay has two bags. The first bag consists of 8 green apples and 3 red apples, and the second bag
consists of 7 red apples and 5 green apples. If two apples are picked, one from each bag, then what is the
probability that both the apples are of different colours?
A. B.

C. D.

E. None of the above


60. Direction: The question given below is followed by two statements, I and II. You have to determine
whether the data given in the statements are sufficient to answer the question, and you have to choose
the best possible answer.
www.byjusexamprep.com

Find the unit digit of a three-digit number.

Statement I: The difference between the original number and the number obtained by reversing its digits is
297

Statement II: The sum of the digits of the three-digit number is 14


A. If the data given in statement I alone is sufficient B. If the data given in statement II alone is sufficient
to answer the question, while the data given in to answer the question, while the data given in
statement II alone is not sufficient to answer the statement I alone is not sufficient to answer the
question question
C. If the data given in either statement I or statement D. If the data given in both the statements together is
II alone is sufficient to answer the question required to answer the question
E. If the data given in both the statements together is
not sufficient to answer the question
61. Find the time taken by pipe Y alone to fill the tank if pipes X, Y, and Z together can fill a tank in 12 hours.

Statement I: Pipes X and Y together can fill the same tank in 16 hours.

Statement II: Pipes Y and Z together can fill the same tank in 18 hours.
A. If the data given in statement I alone is sufficient B. If the data given in statement II alone is sufficient
to answer the question, while the data given in to answer the question, while the data given in
statement II alone is not sufficient to answer the statement I alone is not sufficient to answer the
question question
C. If the data given in either statement I or statement D. If the data given in both the statements together is
II alone is sufficient to answer the question required to answer the question
E. If the data given in both the statements together is
not sufficient to answer the question
62. Direction: Study the following information carefully, and answer the questions given below.

The bar-graph given below shows the total number of students (boys and girls) who study in four different
branches of engineering viz., mechanical, electrical, electronics, and computer science branches of a
college. There are only two types of students in each branch. There are students who want to get a good
job and some others who want to pursue higher studies. The number of students who want to pursue
higher studies is shown in the table.
www.byjusexamprep.com

Note: Assume that there are only two kinds of students, i.e., those who want either a job or those want to
pursue higher studies.

Find the ratio of students in the four branches who want to get jobs to those who want to pursue higher
studies.
A. 5 : 13 B. 5 : 16
C. 7 : 15 D. 7 : 16
E. 15 : 7
63. 30% of students who want to pursue higher studies in the mechanical branch are girls and of the

computer science students who want to do jobs are also girls. Find the difference between the number of
www.byjusexamprep.com

boys in the mechanical branch who want to pursue higher studies and the number of boys in the computer
science branch who want to get jobs.
A. 40 B. 55
C. 50 D. 52
E. 45
64. In another branch, i.e., civil, the total number of students who want to pursue higher studies is 61 % of

the students of the electronics branch who want to get jobs and the number of the students in the civil
branch who want to get jobs is 300% of the students who want to pursue higher studies in the same
branch. No student wants anything other than getting a job or pursuing higher studies in the civil branch.
find the total number of students in the civil branch.
A. 290 B. 293
C. 296 D. 298
E. 299
65. If the ratio of the number of boys to that of girls in the computer science branch is 4 : 3 and if 90% of girls
in this branch want to pursue higher studies while the rest of the girls want to do jobs, then what is the
ratio of the number of boys in computer science who want to get jobs to that of the boys who want to
pursue higher studies?
A. 17 : 40 B. 19 : 45
C. 23 : 48 D. 29 : 51
E. 3 : 5
66. Find the number of three-digit numbers that leaves a remainder of 24 when divided by 26
A. 36 B. 34
C. 33 D. 35
E. 37
67. A solution consists of 25% salt, and the remaining is water. How much percent of water needs to be
evaporated such that the salt becomes 40% of the solution?
A. 20% B. 25%
C. 33.33% D. 40%
E. 50%
68. In a group of 7 friends, there are four boys and three girls. On a particular day, all the boys bought the
same type of book, and all the girls bought the same type of book. Each girl spent Rs. y, and each boy
www.byjusexamprep.com

spent Rs. 98 more than each girl. If the product of the total amount spent by the boys and the total amount
spent by the girls is Rs. 10176, find the amount spent by each girl.
A. Rs. 12 B. Rs. 16
C. Rs. 6 D. Rs. 8
E. Rs. 14
69. A class teacher gave 60 students 3 flavours of ice cream. It is known that 15 of them ate all three flavours,
and 40 of them ate at least two of the three flavours. A total of 110 cups of ice cream was eaten. How
many students did not eat any of the flavours?
A. 5 B. 10
C. 15 D. 20
E. 25
70. The question given below is followed by two statements, I and II. You have to determine whether the data
given in the statements are sufficient to answer the question and you have to choose the best possible
answer.

Find the values of a and b if a < b.

Statement I: A four-digit number 3a6b is divisible by 88

Statement II: A three-digit number a6b is divisible by 12


A. If the data given in statement I alone is sufficient B. If the data given in statement II alone is sufficient
to answer the question, while the data given in to answer the question, while the data given in
statement II alone is not sufficient to answer the statement I alone is not sufficient to answer the
question question
C. If the data given in either statement I or statement D. If the data given in both the statements together is
II alone is sufficient to answer the question required to answer the question
E. If the data given in both the statements together is
not sufficient to answer the question
71. The question given below is followed by two statements, I and II. You have to determine whether the data
given in the statements are sufficient to answer the question, and you have to choose the best possible
answer.

Find the value of ‘x’.


www.byjusexamprep.com

Statement I:

Statement II:
A. If the data given in statement I alone is sufficient B. If the data given in statement II alone is sufficient
to answer the question, while the data given in to answer the question, while the data given in
statement II alone is not sufficient to answer the statement I alone is not sufficient to answer the
question question
C. If the data given in either statement I or statement D. If the data given in both the statements together is
II alone is sufficient to answer the question required to answer the question
E. If the data given in both the statements together is
not sufficient to answer the question
72. Using a given set of ‘n’ points, the number of triangles that can be formed is 84. If none of the ‘n’ points
are collinear, find the number of triangles that can be formed using 2n points.
A. 168 B. 464
C. 682 D. 816
E. 644
73. Which of the following should come in place of ‘?’ in the series given below?
E10 G21 I36 K55 ?
A. N12 B. M78
C. J13 D. O75
E. None of the above
74. Which of the following numbers will complete the puzzle?
www.byjusexamprep.com

A. 29 B. 31
C. 33 D. 34
E. 35
75. Direction: In the following question, there is a statement followed by some courses of actions. Assuming
everything in the statement to be true, select the most appropriate option.

Statement: The social activists in the city of Highgarden will go on an indefinite hunger strike to protest
against the city council’s proposal to sell wine in supermarkets and walk-in stores across the city at a flat
annual licensing fee of 5,000 gold coins.

Courses of action:

I. The council should increase the annual licensing fee by three times.

II. The council should roll back the plan and invite suggestions from citizens.
A. If only I follows B. If only II follows
C. If neither I nor II follows D. If both I and II follow
E. Either I or II follows
76. Statement:

The students of classes I to V at The Smile School are reluctant to come to the school after the pandemic
and want all the classes to happen online.
www.byjusexamprep.com

Courses of action:

I. The school authorities can hold special sports and fun activities in schools.

II. The school authorities should threaten the students that they won’t be allowed to appear for the annual
exams if they do not attend offline classes.
A. Only course of action I follows. B. Only course of action II follows.
C. Either I or II follows. D. Both I and II follow.
E. Neither I nor II follows.
77. Statement:

The heavy overnight rain has led to several areas in South Chennai going without power supply due to
cable faults, feeder trappings, and shutting down as part of precautionary measures.

Courses of action:

I. The local MCD should try to resume power supply via safe alternative mediums as the rains are very
likely to continue for the next few days.

II. The government should provide a dedicated helpline number to address power-related grievances.

III. Power supply should be resumed immediately as the local industry’s production will be severely
affected.
A. Only I and II follow. B. Only II and III follow.
C. Only I and III follow. D. Only I follows.
E. Either I or II follows.
78. Statement:

Lately, a lot of people have been diagnosed with digestive issues. There are certain things that Ayurveda
doesn't recommend you to do immediately after having lunch, breakfast, or dinner. As per the ancient
medicine system, drinking water after meals or going to bed right after dinner is not a good idea and can
burden your digestive system.

Course of action:
www.byjusexamprep.com

I. Avoid any activity after having dinner and take rest as the circulation of blood and nerve responses is
oriented towards the stomach and intestines to aid the digestion process.

II. There should be a public awareness camp to educate people about the correct practices that should be
adopted for reducing the burden on the digestive system.
A. Only I follows. B. Only II follows.
C. Neither I nor II follows. D. Both I and II follow.
E. Either I or II follows.
79. Direction: Study the following information carefully, and answer the given questions.

When an arrangement machine is given a line of numbers and words, it rearranges them according to a
particular logic at each step as shown below.

Input: rose yellow 40 bag 45 music fish 25 15 30

Step I: 15 rose yellow 40 bag 45 fish 25 30 music

Step II: 25 15 yellow 40 bag 45 fish 30 music rose

Step III: 45 25 15 yellow 40 fish 30 music rose bag

Step IV: 30 45 25 15 yellow 40 music rose bag fish

Step V: 40 30 45 25 15 music rose bag fish yellow

Step V is the last step of the above input.

As per the rules followed in the steps given above, find out the steps for the following input:

Input: butter range 20 pillow sick 25 35 rich 45 40

Which of the following will be at the rightmost position in Step IV?


A. rich B. butter
C. pillow D. range
E. sick
80. Which number will be at the leftmost position in Step V?
www.byjusexamprep.com

A. 20 B. 25
C. 35 D. 40
E. 45
81. Which of the following will be at the sixth place from left in Step III?
A. 20 B. 40
C. 25 D. rich
E. pillow
82. What will be the step number of the following arrangement of words and numbers according to the logic
being followed?

45 35 25 butter 20 pillow 40 range rich sick


A. Step III B. Step II
C. Step IV D. Step V
E. None of the above
83. Direction: Given below is a statement followed by two arguments. Consider the statement to be true and
figure out whether the given arguments are strong or not.

Statement: Should the use of animals in movies be stopped?

Arguments:

I. Yes, there have been many reported instances in which the animals that feature in movies have been
traumatised due to the atrocities inflicted on them by their handlers.

II. No, animals are an intrinsic part of social life and, thus, should be included wherever required.
A. Only I is strong. B. Only II is strong.
C. Either I or II is strong. D. Neither I nor II is strong.
E. Either I or II is strong.
84. Statement: Should the government spend more money on education than on defence?

Arguments:

I. No, defence should be of prime importance. Only when a country’s borders are secure can civilians
have a peaceful life.
www.byjusexamprep.com

II. No, not necessarily as it depends completely on the requirements of the country and its priorities,
keeping in mind what is best for its citizens.
A. Only I is strong. B. Only II is strong.
C. Either I or II is strong. D. Neither I nor II is strong.
E. Either I or II is strong.
85. Statement: Should fire safety systems and training of people in fire safety be made compulsory in all high
rise buildings?

Arguments:

I. No, the number of fires occurring in high rise buildings is miniscule.

II. No, it will be an additional cost that residents of high-rise buildings will be forced to bear.
A. Only I is strong. B. Only II is strong.
C. Either I or II is strong. D. Neither I nor II is strong.
E. Either I or II is strong
86. Direction: Read the following information carefully, and answer the questions given below.

Eight persons (L, M, N, O, P, Q, R, and S) like eight different brands, i.e., Gogole, Papple, Tita, Zipro, OG,
Xord, Pony, and Tara but not necessarily in the same order. Each of them has a different age.

The one who likes Papple is neither the youngest nor the oldest. N is older than the one who likes Gogole.
The one who likes Pony is younger than Q, and at least one person is younger than the one who likes
Pony. S is younger than P and someone else as well. P is older than the one who likes Tara, who in turn is
older than the one who likes Gogole. R is just younger than Q. Q and N are younger than the one who
likes OG, who is younger than the one who likes Tara and S. The one who likes Zipro is just older than the
one who likes Xord, who is younger than the one who likes OG. N does not like Papple. L is older but not
immediately older than M, who is older than Q. The one who likes Tara is not the oldest. There are as
many people who are younger than R as those who are older than S.

Which of the following brands are liked by the oldest and third youngest persons respectively?
A. Pony and Xord B. Gogole and OG
C. Tara and Zipro D. Tita and Xord
E. Gogole and Zipro
www.byjusexamprep.com

87. How many people are older than the one who likes Papple?
A. None B. One
C. Two D. Three
E. More than three
88. Which of the following statements is true about M?
B. M is older than R, and M’s age is closest to that of
A. M is the second oldest among the eight persons.
S among all people younger than S.
C. M is just older than N and O. D. M likes Gogole.
E. None of the above
89. Whose age amongst the following is closest and lower to that of the one who likes Tara?
A. L B. The one who likes Papple
C. R D. The one who likes Tita
E. The one who likes OG
90. Direction: In the following question, two statements are followed by two conclusions numbered I and II.
Assume the two statements to be true even if they are at variance with commonly-known facts. Then pick
the correct answer from the choices given below.

Statements:

Some markets are industries.

Some industries are sectors.

Conclusions:

I. Some industries are not sectors.

II. No markets are sectors.


A. Only conclusion I follows. B. Only conclusion II follows.
C. Both conclusions I and II follow. D. Neither conclusion I nor conclusion II follows.
E. Either I or II follows.
91. Direction: Study the following data carefully, and answer the questions accordingly.
www.byjusexamprep.com

Six different dishes are made for dinner at night in a girls’ hostel’s mess on six different days, from Sunday
to Friday of the same week. These dishes are made by six different cooks, i.e., Q, R, S, T, U, and V but
not necessarily in the same order.

V made the dish after Monday but did not make chole. Exactly one dish is made in the period between
which biryani and the dish that is made by Q are prepared. R made the dish after Monday but did not
make shahi paneer nor rajma chawal. S made the dish (which is not korma) after rajma chawal but not on
the next day. U did not make biryani or shahi paneer. Lemon rice was made immediately before biryani,
which was not made by S. Exactly two dishes are made between shahi paneer and the dish made by S.
Rajma chawal is made after Tuesday.

How many dishes are made before chole?


A. Three B. One
C. Four D. Two
E. Five
92. Who amongst the following made biryani?
A. The one who made the dish on Friday B. V
C. The one who made the dish on the day
D. U
immediately before Tuesday
E. R
93. Find the odd one out amongst the following options.
A. Korma: Friday B. Shahi paneer: Wednesday
C. U: T D. V: S
E. Rajma chawal: Korma
94. Which of the following dishes is made on Sunday?
A. Chole B. Shahi paneer
C. Lemon rice D. Biryani
E. None of the above
95. Direction: Consider the following statements:

Fruits are one of the healthiest food options out there.


www.byjusexamprep.com

Fruits consist of various nutrients such as vitamins, potassium, fibre, etc.

Besides, fruits reduce dependence on artificial sugar.

Which one of the following inferences can be drawn from the above statements?
B. Fruits and vegetables should be the primary
A. Fruits are the best food option.
constituent of a person’s diet.
C. Eating fruits can help one in leading a healthy
D. Fruits create the backbone of a heavy diet.
lifestyle.
E. Fruits can take care of various diseases.
96. Direction: Read the following passage and answer the given question:

The Russia-Ukraine war has seen the rise of the prices of natural resources such as petroleum and
natural gas.

Which of the following can be inferred from the above statement?


A. The Russia-Ukraine war might have led to a B. The rising prices of fossil fuels will lead to
massive shortage of fossil fuels. catastrophic economic consequences.
C. The war has seen Russia being handed strict D. The war will cost Russia almost twice as much as
sanctions by the western nations. the second world war.
E. The Russia-Ukraine war has not been a win-win
scenario for both the countries.
97. Direction: Read the following passage and answer the given question:

It has been observed in recent times that despite whatever the reviews have to say, if a movie strikes a
chord with the audience, then it is bound to succeed.

Which of the following can be concluded from the above statement?


A. Any movie that is hugely entertaining will
B. Any movie that is inspirational will succeed.
succeed.
C. Movies that the audience can relate to are bound D. Movies that have big movie stars are bound to
to succeed. succeed despite their content.
E. Movies need to have solid content for them to
succeed.
www.byjusexamprep.com

98. Direction: Read the following passage and answer the given question:

A tomb with a warning written in blood red text, which warns against opening it, has been discovered in
Israel. The news has been gaining traction on social media since the time the images of the tomb surfaced
online.

Which of the following can be concluded from the given information?


A. The tomb must not be opened otherwise evil may B. The tomb is likely to have existed from mediaeval
befall on us all. times.
C. The tomb is gaining attraction because it is quite D. The tomb is not the last one that can be
unusual. discovered in Israel.
E. The tomb is no longer a hidden gem.
99. Direction: Read the following passage and answer the given question:

A study published in a scientific journal has rung alarm bells for Indian megacity Mumbai. The study says
that the coastal city is sinking into the sea.

Which of the following can be inferred from the given information?


A. Mumbai is sinking because there are various B. Mumbai’s sinking has nothing to do with global
Tsunamis expected this year. warming.
C. Mumbai needs to address the issue of preserving D. Nothing can stop Mumbai from submerging in the
its coastal areas from submerging in the sea. sea.
E. Mumbai’s coastal areas are in danger of being
submerged in the sea.
100. Directions: Study the following data carefully, and answer the questions accordingly.

A certain number of people are sitting around a circular table at equal distances from each other and are
facing the centre. There are two people between E and F. G sits at fourth place to E's left. One person sits
between F and I. I and G are not neighbours. E and I are not neighbours. J sits fourth to the right of K.
Two people sit between K and I. G sits to the immediate left of J. How many people will be there if 5 new
people decide to join the table?
A. 20 B. 15
C. 19 D. 14
E. 21
www.byjusexamprep.com

101. The following diagram represents the number of students who are good in different subjects. How many
students are good in both mathematics and sociology but not good in accountancy?

A. 7 B. 10
C. 5 D. 9
E. 12
102. Direction: In each of the questions below are given a few statements followed by two conclusions. You
have to take the given statements to be true even if they seem to be at variance from commonly known
facts. Read all the conclusions and then decide which of the given conclusions logically follow from the
given statements disregarding commonly known facts.

Statements:
Some polymers are clay.
Only a few slates are ink.
Some slates are clay.
Conclusions:
I. Some ink can be polymer.
II. All ink is not clay.
A. If conclusion I follows B. If conclusion II follows
C. Either conclusion I or conclusion II follows. D. Neither conclusion I nor conclusion II follows
E. Both the conclusions follow
103. Direction: Read the following passage and answer the given question:

To bring down the water pollution levels in Delhi, a big goal was announced by the government a couple of
months ago that the city would be able to process 95 per cent of the wastewater by the end of 2022. This,
www.byjusexamprep.com

according to the government, was in sync with plans to reduce the pollution levels in the Yamuna River.

Which of the following can be concluded from the given excerpt?


A. The levels of water and air pollution levels in Delhi B. The government of Delhi is serious about
go hand-in-hand. reducing pollution levels in Delhi.
C. The government has plans to eliminate water D. Pollution levels in Yamuna need a huge amount of
pollution from Yamuna River by the end of 2022 processing.
E. The government is likely to come up with a major
update on the level of pollution in Yamuna River.
104. Direction: Read the following passage and answer the given question:

Governments around the world are developing strategies and suites of policies to support their climate
change mitigating ‘net-zero’ ambitions. Of course, more recently, linked to the terrible Russian war against
Ukraine, policymakers are also looking to limit exposure to fossil fuel imports and the risks they pose. The
current US administration has described hydrogen as a ‘game-changer’ in the fight against climate change
and the transition away from fossil fuels. The UK Prime Minister Boris Johnson believes it ‘has perhaps
the greatest potential of all’, while Ireland’s Tánaiste (the deputy head of government) has called it ‘the
holy grail’ of energy policy.

Which of the following can be a valid assumption in the given context?


B. Countries across the world are looking to
A. Hydrogen is the future of our energy needs. incorporate usage of Hydrogen to fulfil their energy
needs.
C. Given its abundance, Hydrogen can easily fulfil
D. Hydrogen will emerge as the biggest source of
energy requirements for almost every country
energy in the future.
around the world.
E. Policymakers are looking to cut their usage of
fossil fuels.
105. Direction: Read the following passage and answer the given question:

Research has found that mental illness likely has a genetic component, but that mental illness is most
probably caused by a combination of genetic and environmental components. What’s more, certain mental
health disorders—such as bipolar disorder, schizophrenia, and depression—are more closely tied to
www.byjusexamprep.com

genetics than other disorders. Studies looking at the connections between genetics and mental illness are
ongoing, and there’s still much to be learned.

Which of the following is a valid assumption in the given context?


A. Mental illness being caused by a genetic
B. Genetics play a huge role in determining whether
component is dependent upon the type of disease
one will suffer from mental illness in the future or not.
one is suffering from.
C. There is still no clarity over whether there is any
D. People having genetic combinations that are awry
connection between mental illness and genetic
are more likely to suffer from mental illnesses.
components.
E. The external stimulus one is provided also plays a
huge role in determining one’s mental health.
106. Direction: Read the following passage and answer the given question:

The commonwealth games, which began in Birmingham recently, suffer from a Pepsi Cola problem. The
athletic standard is among the highest in international sport. But, like the fizzy drink, the games are
overshadowed by a bigger rival. The summer Olympics are often unwieldy and expensive but are
guaranteed to draw an audience of billions. The Commonwealth Games have the same drawbacks. But
they are also at risk of becoming irrelevant.

Which of the following, if true, would strengthen the claim that commonwealth games are not becoming
redundant?
A. The public turnout over the course of the B. The Commonwealth games have failed to attract
commonwealth games has been terrific. big money sponsors.
C. The Commonwealth games run the risk of being D. The Commonwealth games authorities need a
compared to the Olympics. fresh perspective on how to revive games.
E. The Commonwealth games are still up there with
the best of inter-country competitions.
107. Direction: Read the following passage and answer the given question:

PM Modi’s Independence Day speech set an ambitious target—India must become a developed country
by 2047. ‘Developed’ is as elastic a term as ‘developing’, and economists can provide multiple criteria. But
income, and per capita income, is certainly one key criterion, and the other is that non-farm employment
must be far more than farm employment. The two criteria are linked—incomes rise as the share of people
www.byjusexamprep.com

in farming go down and manufacturing and services jobs multiply. This has held true from the beginning of
the Industrial Revolution in Britain around 1760. That was the most consequential change in human
society after the beginning of agriculture.

Which of the following, if true, would strengthen the idea that farm employment is not a hindrance to India
becoming a developed country?
B. Farm employment provides low-skilled people
A. Non-farm employment has increased in recent
with jobs, something that sectors such as
times, despite the rise in income in the farm sector.
manufacturing can’t provide.
C. India is a farm-oriented country and no amount of
D. Farm employment has sustained the lives of
growth in other sectors will aid in the decline in farm
millions of people since our independence.
income.
E. Farm employment has provided sustenance to
people since ages.
108. Which of the following numbers will replace the question mark in the figure below?

A. 11 B. 7
C. 21 D. 23
E. 15
www.byjusexamprep.com

Solutions

1. B
Sol.
Only sentence B is correct. The reasons are as follows:

A: ‘among’ is used when the comparison is between more than 2 nouns. In the given case, the
comparison is between exactly two nouns (men's and women's). In such cases, we use
'between'. It should be 'between' instead of 'among'.

C: When we are stating multiple terms to be joined by the preposition ‘and’, it is used only
once, before the last term. In the given case, there are three verbs depicting the actions of
onlookers (shouting, cheering, waving). We should use ‘and’ only once, just before waving. It
should be ‘The onlookers were shouting, cheering, and waving banners.’

Hence, the correct answer is option B.

2. D
Sol.
Both options A and B are correct. In option C, ‘any time’ is an adverb and should not be
preceded by ‘the’.

Hence, the correct answer is option D.

3. A
Sol.
Since the first sentence is talking about the government’s recent interventions being effective,
we need to talk about some of those interventions in the next statement, which would be
statement A. It will be followed by B, which explores government decisions even more by
talking about the reduction in fuel prices, which has shown the desired effect. Now, only
options A and C have AB as a pair at the beginning. So, the rest of the options are rejected.
www.byjusexamprep.com

Now, the presence of the word ‘this’ in statement D indicates that it needs to be preceded by
something. So, C would come before D. So, the correct answer is ABCD.

Hence, the correct answer is option A.

4. D
Sol.
Here, all you need to do is figure out the statement succeeding statement A, and you will
arrive at the answer. Now, if you look at the jumbled up statements closely, it is clear that D
and B form a mandatory pair. D is talking about the visit of the US commander of Army Pacific,
while B talks about their opinions on China-India relations and China’s activity in Ladakh. Also,
C should come after D-B because of the presence of the word ‘then’. Hence, ADBC is the
correct sequence.

Hence, the correct answer is option D.

5. A
Sol. The meaning of the difficult words given in the answer options are as follows:

Stipulate means to specify something as a condition or requirement in a contract or agreement


or to make an express demand.

Procure means to get something with special effort.

Sustenance means something that gives support, endurance, or strength.

Consign means to commit irrevocably or commit forever.

To volunteer means to agree freely.

A command is an order, whereas a request is an insistence. Similarly, to stipulate is to specify


something, whereas to propose means to recommend. Thus, option A shows a similar
relationship as the given pair. Option B does not align with the given relationship. Option C can
also be eliminated as committing forever and handing something over, again, does not show
the relationship given in the question. Option E does not make any sense here.
www.byjusexamprep.com

Hence, the correct answer is option A.

6. C
Sol. The meaning of the difficult words given in the answer options are as follows:

Curtail means to cut something short.

Ravenous means having a huge appetite.

Appetite means the need or desire for food or a strong wish for something.

Impeccable means flawless.

Captivating means charmingly or irresistibly appealing.

Syncopate means to make less in extent or duration.

Kempt: (of a person or a place) maintained in a neat and clean condition; well cared for

A boundless thing does not have any limit. Boundless and limit are antonyms. Similarly,
something impeccable or flawless does not have any flaw. Impeccable and flaw are also
antonyms.

A curtailed piece may still have length, and it is impossible for something real to not have a
length. A ravenous person has an appetite. Similarly, a captivating person or thing is appealing
and has your interest. So, options B and D can be eliminated. Option E does not consist of
words having any relationship at all.

Hence, the correct answer is option C.

7. A
Sol.
Only sentence A is correct. The reasons why B and C are incorrect as follows:

B: ‘infer’ is used when we draw a conclusion from a given observation. Here, the observation
is ‘the lack of reply to our letters’. But the sentence suggests that this observation itself is
www.byjusexamprep.com

drawing the conclusion of ‘lack of interest’. This is a logical error. Observations can imply
certain conclusions. So, it should be ‘imply’ instead of ‘infer’.

C: When we use ‘both’ with the ‘be’ verb, it is placed after the verb. It should be ‘They were
both widows…’

Hence, the correct answer is option A.

8. C
Sol.
Only sentence C is correct. The reasons are as follows:

A: The sentence is in a passive voice. We should use the verb ‘have been’ before the main
verb (isolated).

B: The auxiliary verb ‘be’ should be followed by the past participle form of the main verb. It
should be ‘taken’ instead of ‘took’.

Hence, the correct answer is C.

9. D
Sol.
All the given options are incorrect. The reasons are as follows:

A: The first part of the sentence uses the verb in simple present tense (fill), while the second
part uses the verb in simple past tense (could). This is a grammatical error. Both must follow
the same tense.

B: ‘beside’ is used to indicate the physical space next to someone or something. ‘Marriage
ceremony’ is not capable of having such a physical space next to it. It should be ‘aside’, which
will indicate money saved for the marriage ceremony.

C: ‘Many of the’ is used to indicate one among many. So, it should be ‘villages’, instead of
‘village’ to show the ‘many’.
www.byjusexamprep.com

10. B
Sol.
The purpose behind putting such a large sentence is to scare off the aspirants, but one always
needs to make sure they at least give the question a try. This question is actually quite easy
because all you need to do is to figure out the opening statement. And that is also quite easy if
you follow the first statement closely. Now, refer to these lines:

‘The resignation of Sri Lanka’s Finance Minister Basil Rajapaksa as Member of Parliament on
Thursday did not come as a surprise, given the adverse…’ In A, the author is talking about
Basil Rajapaksa becoming an MP in Sri Lanka and why his ouster was not surprising. Now,
statement D perfectly brings out what was actually surprising in the given scheme of affairs.
Hence, it follows A.

Hence, the correct answer is option B.

11. B
Sol.
The first statement introduces us to the multi-stage monsoon forecast system along with IMD’s
projections. It is then followed by statement B which talks about the April forecast and its
different aspects. So, statements A and B are connected to one another.

Statements B-C form a pair. C introduces us to another topic of discussion after running the
course with the April forecast, i.e., its limitations in forecasting whether the rain will be lopsided
or evenly distributed geographically.

C-D form a pair. C talks about the limitations of the April forecast, and D, with ‘Thus’, extends
that idea by stating that the April forecast is of little utility value.

Hence, the correct answer is option B.

12. D
Sol.
The given sentence is grammatically and contextually correct. Note that the phrase 'but it turns
www.byjusexamprep.com

out' is generally used for saying what the situation really is.

Hence, option D is the correct answer.

13. A
Sol.
The usage of ‘has prepared to’ is wrong and ‘was prepared to’ will be appropriate instead of
that. The sentence is in the form of reported/indirect speech. Since the reporting verb ‘said’ is
in the past form, the reported speech will also be in the past tense. Thus, option A is the
correct answer.

Hence, the correct answer is option A.

14. A
Sol. The meaning of the difficult words given in the answer options are as follows:

Slough means to cast off hair, skin, horn, or feathers.

Overwinter means to last through or pass the winter.

Shuck means to remove from the shell.

Oysters are marine molluscs with rough or irregular shells.

Moulting is to cast off hair, skin, horn, or feathers. A bird moults its feathers, and a snake
sloughs its skin. None of the other options shows a similar relationship.

Hence, the correct answer is option A.

15. D
Sol. The meaning of the difficult words given in the answer choices are as follows:

Acquit means to set someone free.

Quarry refers to an animal that is hunted or killed; prey

Defile means to treat a sacred place or object shamefully or with great disrespect.
www.byjusexamprep.com

Recriminate means a retaliatory accusation.

Redeem means to buy back.

Rescind means to take away or cancel or remove.

Defame: damage the good reputation of (someone); slander or libel

Acquit: free (someone) from a criminal charge by a verdict of not guilty

Renounce means to give up on a position or authority. Pledge is to stand on a prior


agreement. Hence, the words are antonyms. Similarly, rescinding is cancelling an order.
Rescind and order show an antonym relationship as well. Options B and C can be eliminated.
The pair of words in option A are not antonyms.

Hence, the correct answer is option D.

16. B
Sol.
The preposition ‘of’ must be used as it is used to denote belongingness. Hence, ‘of’ must be
used to complete the sentence correctly. Therefore, option B is the correct answer.

17. C
Sol.
The word ‘admission’ is always followed by ‘to’ rather than ‘in’. The preposition ‘to’ indicates
approaching or reaching a particular condition. Here, the condition is getting access to the golf
course.

Hence, option C is the correct answer.

18. D
Sol.
Here, the preposition ‘but’ is used because the phrase ‘all but’ means ‘very nearly’, and it
makes the sentence sensible and logical. According to the context, we can infer that the prince
has nearly gathered absolute power.
www.byjusexamprep.com

Hence, the correct answer is option D.

19. C
Sol. Passage summary: The article talks about the growing stock of cryptocurrency in India as the
author highlights the need to provide a solid legal framework to the industry.

Genre: Cryptocurrency

Number of words: 475

Type of question: Direct/Fact-based question

In the first paragraph of the passage, the underlying numbers regarding the cryptocurrency
industry are mentioned. The numbers clearly indicate that the cryptocurrency industry is on the
rise. Refer to the extract: ‘The Indian crypto asset industry has witnessed exponential growth
over the last five years.’

On the other hand, the rest of the answer options talk about one particular prospect of the
cryptocurrency industry. Even that being true is not possible as per the passage because we
don’t know much about these scenarios. Hence, option C is the correct answer.

20. B
Sol. Passage summary: The article in hand talks about the growing stock of cryptocurrency in India
as the author highlights the need to provide a solid legal framework to the industry.

Genre: Cryptocurrency

Number of words: 475

Type of question: Specific idea based

The author believes that if crypto represents our future, then it needs to be provided with legal
assistance to make sure it reaches the place currently occupied by the big tech companies in
India. The author believes that crypto needs a legal framework that will guarantee it a legal
status in India and, hence, will foster growth in the technology. This idea is best captured in
option B.
www.byjusexamprep.com

Hence, option B is the correct answer.

21. C
Sol. Passage summary: The article in hand talks about the growing stock of cryptocurrency in India
as the author highlights the need to provide a solid legal framework to the industry.

Genre: Cryptocurrency

Number of words: 475

Type of question: Main idea based

Throughout the passage, the author has been concerned with talking about regulating the
cryptocurrency industry in India. The author has discussed various aspects of the issue and
has discussed under what considerations the industry should be regulated in the given market
scenarios. All these ideas suggest that option C is the correct answer here.

Here, the biggest distinction between the correct answer and the incorrect answers is that the
incorrect options tend to favour cryptocurrency in one way or the other. Option A, for instance,
talks about the need to legalise cryptocurrency in India, which means the author is in favour of
legalising cryptocurrency. However, the author has not declared his/her/their penchant towards
cryptocurrency. The author has simply discussed cryptocurrency and its various aspects
without supporting its cause.

Hence, the correct answer is option C.

22. A
Sol. Passage summary: The article in hand talks about the growing stock of cryptocurrency in India
as the author highlights the need to provide a solid legal framework to the industry.

Genre: Cryptocurrency

Number of words: 475

Type of question: Inference-based question


www.byjusexamprep.com

The passage states that the study has mentioned various scenarios that can be used to come
up with a concrete regulatory framework for cryptocurrency in India. While going through all
the scenarios, the author has mentioned their pros and cons and also stated the different
scenarios that can pan out in the case of India. However, the author has not talked about a
particular scenario that is likely to be the one that concretely provides us the solution to the
problem at hand. So, all we can say is that the regulatory framework to provide legal status to
the cryptocurrency industry in India needs to be fine-tuned from all the different cases and
from our market conditions in today’s times.

Hence, the correct answer is option A.

23. D
Sol. Passage summary:

The passage describes the blossoming of ties between India and the USA since the end of the
Cold War due to various reasons. These close ties rest on four mutually reinforcing pillars, of
which one, i.e., liberal democracy in India and the USA, is under increasing threat.

Genre: International relations / Politics

Number of words: 508

Type of question: Direct/Fact-based question

‘There were three principal drivers of this shift. First, the collapse of the communist model and
India’s embrace of globalisation and market economics in 1991 reoriented India’s focus
westward. Second, India’s emergence as a nuclear-armed power and America’s willingness to
incorporate India into the global civil nuclear regime, despite the fact it is not a signatory to the
Non-Proliferation Treaty (NPT), brought the countries closer together. Third, China’s rise as a
regional power with global aspirations…’ Options A, B, and C are included in the extract.

Hence, the correct answer is option D.

24. C
www.byjusexamprep.com

Sol. Passage summary:

The passage describes the blossoming of ties between India and the USA since the end of the
Cold War due to various reasons. These close ties rest on four mutually reinforcing pillars, of
which one, i.e., liberal democracy in India and the USA, is under increasing threat.

Genre: International relations

Number of words: 508

Type of question: Direct/Fact-based question

Refer to the lines: ‘Unfortunately for the two nations, there is an accumulating body of
evidence that suggests that at least one of the aforementioned pillars—shared liberal
democratic commitments—can no longer be taken for granted.’ This extract is followed by
‘Recent anxieties about democratic backsliding across the globe have not left India untouched.
This is not to deny that American democracy too is at a nadir.’ Therefore, both India and the
USA have ceased to be liberal democracies. This makes option C the correct answer.

Options A and B are not mentioned in the passage.

Option D is incorrect. Migration levels of Indians to America are high and not low. Option E is
completely out of scope here.

Option E is too extreme to be considered as a valid answer.

Hence, the correct answer is option C.

25. B
Sol. Passage summary:

The passage describes the blossoming of ties between India and the USA since the end of the
Cold War due to various reasons. These close ties rest on four mutually reinforcing pillars, of
which one, i.e., liberal democracy in India and the USA, is under increasing threat.
www.byjusexamprep.com

Genre: International relations

Number of words: 508

Type of question: Direct/Fact-based question

Refer to the lines: ‘These threats to American democracy are real and visceral. But America’s
struggles are arguably of a lesser magnitude than India’s…’ Thus, option B is the correct
answer.

Options A and D have not been mentioned in the passage.

Option C is incorrect. America’s democratic downslide is real.

Option E is far too extreme.

Hence, the correct answer is option B.

26. A
Sol. Passage summary:

The passage describes the blossoming of ties between India and the USA since the end of the
Cold War due to various reasons. These close ties rest on four mutually reinforcing pillars, of
which one, i.e., liberal democracy in India and the USA, is under increasing threat.

Genre: International relations

Number of words: 508

Type of question: Tone question

Refer to these lines: ‘But America’s struggles are arguably of a lesser magnitude than India’s
given the relative strength of its institutions, elevated per capita income, and democratic
longevity.’ The author is not pessimistic about America’s democracy downslide but is in
contrast pessimistic about India’s since by default, he implies that institutions in India are not
as strong nor is the per capita income very high.
www.byjusexamprep.com

Thus, the correct answer is option A.

Options B, C, D, and E are eliminated by the explanation given above.

Hence, the correct answer is option A.

27. D
Sol.
Passage summary: The article talks about the CUET and the need for CUET to focus on
vocational subjects to facilitate the comprehensive development of students.

Genre: Education

Number of words: 424

Type of question: Main idea-based question

Here, the author has primarily written the article against the backdrop of the CUET exam
which did not give adequate importance to vocational subjects. The author has cited the views
of various educators who believe that focussing on only a select vocational subjects would
reduce the importance of vocational education and consequently, would not align with the New
Education Policy and its aim to make students achieve excellence in all fields of life. This idea
is best captured in option D, and hence it is the right answer.

The rest of the answer options do not keep CUET in focus, and hence these are rejected.

Hence, the correct answer is option D.

28. B
Sol.
Passage summary: The article talks about the CUET and the need for CUET to focus on
vocational subjects to facilitate the comprehensive development of students.

Genre: Education
www.byjusexamprep.com

Number of words: 424

Type of question: Specific idea-based question

Refer to this line:

‘Still, at its starting point, stakeholders are more concerned about figuring out finer details of
CUET such as correlating marks with courses, than focus on including vocational subjects.’

From these lines, it is clear that CUET is focusing more on figuring out the finer details of
CUET instead of including more vocational subjects. Hence, option B is the right answer here.

Option C is incorrect because it is the byproduct of CUET not focusing on vocational subjects.
Option A is also mentioned as a byproduct of limited students and lesser interest from the
CUET conducting body, whereas option D is not mentioned in the passage. Option E is false
under the given context.

Hence, the correct answer is option B.

29. C
Sol.
Passage summary: The article talks about the CUET and the need for CUET to focus on
vocational subjects to facilitate the comprehensive development of students.

Genre: Education

Number of words: 424

Type of question: Specific idea-based question

Refer to this line from the third paragraph of the passage:

‘Gradually, NEP’s aim to remove focus from attaining only academic excellence and making
students job-ready will remain unfulfilled.’
www.byjusexamprep.com

Here, the author is talking about NEP’s ultimate objective, which is to make students job-ready
instead of achieving only academic excellence. Now, if the NEP is intent on rising beyond
academic excellence, it means it is focused on opening the doors for a variety of career
options. This idea is best captured in option C, and hence it is the right answer here.

Here, options B and D can be eliminated straightaway because both are not within the context
of the passage. Option A is eliminated because we can’t say that academic excellence puts
shackles on us. Hence, it is a misrepresentation of the points given in the passage. Option E is
incorrect. The idea is not only to educate but also to open the path for the students.

30. A
Sol.
Passage summary: The article talks about the CUET and the need for CUET to focus on
vocational subjects to facilitate the comprehensive development of students.

Genre: Education

Number of words: 424

Type of question: Inference-based question

Throughout the passage, the author has talked about various problems that have marred the
implementation of vocational subjects’ study in schools. From CUET not paying much
attention, lack of interest from students to schools facing different challenges, we can say that
the implementation of vocational subjects’ study in school is a multi-aspect problem. Thus,
option A is the correct answer.

Hence, the correct answer is option A.

31. D
Sol. The meaning of the difficult words given in the answer options are as follows:

Realism means the state of being actual or real.


www.byjusexamprep.com

Aesthetic means relating to or dealing with beauty.

Intuition means an impression that something might be the case.

Empirical means something derived from an experiment or observation rather than a theory.

Inundation refers to an overwhelming abundance of people or things.

Since they did not do any research to build the model, they must have made it from their
intuition, guesswork, experience, or inference. The second part of the sentence suggests that
the model turned out to be correct from a research perspective as well. The ‘but’ in the
sentence indicates it. So, it has to be research-backed or empirically true. Options B and A
can be easily eliminated. Since we are talking about statistics models, empirical will be more
apt than scientific.

Hence, the correct answer is option D.

32. B
Sol. The meaning of the difficult words given in the answer options are as follows:

Delinquent usually means a young person who performs illegal or immoral acts.

Frail means something that is easily broken or fragile.

Valiant means feeling or displaying no fear by temperament.

Dread refers to the suspicion or fear of future harm or misfortune.

Absurd means something unreasonable or unsound.

Hilarity means a mood characterised by high spirits and amusement.

Veneer refers to a layer or covering of something.

If the gloss of rationality of humans is easily breached, then the first blank will accommodate a
word with a negative connotation about human beings. Thus, frail, which is the only negative
word in the first blanks, is the correct choice.
www.byjusexamprep.com

Fears will be followed by another similar word with a negative connotation for the second
blank. Fears and insecurity is the correct match.

Hence, the correct answer is option B.

33. C
Sol. The meaning of the difficult words given in the options are as follows:

Grandiloquent means high flown style.

Historical fiction must serve as a decent genre, work, or category for readers. The rest of the
options do not fit. Now, option C will fit the second blank accurately. Although it is a decent
genre, still people feel it has deviated from both history and fiction.

Option D can be easily eliminated. And between options B and C, C will be more apt for the
blank.

Hence, the correct answer is option C.

34. B
Sol. Three reasons are given for the cheetah’s disappearance in India. These three reasons
together form a combination.
The first sentence talks about the cheetah’s disappearance in India. This means that the
cheetah became extinct.
Hence, the correct answer is option B.

35. B
Sol.
The meaning of the difficult words given in the answer options are as follows:

Fabricate: concoct something artificial or untrue

Flippant: light-minded

Naive: innocent
www.byjusexamprep.com

Gullible: naïve and easily tricked

Caustic: bitter, sharp

Dapper: refers to someone sharply dressed

Craven is the one who lacks courage. So, craven is a coward.

Similarly, a gullible person is naive. The relationship is that of synonyms.

A fabricator may not necessarily be a flippant. The relationship is not synonymous.

Opponent, again, may not always be caustic. Take the example of sports teams who usually
play without being caustic. So, option C need not be necessarily true.

Judge and Impartial are not synonyms. Eliminate option D.

Option E does not have words with any relation.

Hence, the correct answer is option B.

36. D
Sol.
The meaning of the difficult words given in the answer options are as follows:

Sterilise: To make powerless or useless by restraining from a normal function, relation, or


participation

A meaning of enfranchise is to grant voting rights. So, a person who has voting rights is
enfranchised. Similarly, by illumination, by granting or supplying light, we can see. Or a thing
which can be seen is illuminated.

Selling may take place without promotion as well. Options B, C, and E can be easily
eliminated.
www.byjusexamprep.com

Hence, the correct answer is option D.

37. B
Sol. The table given below shows the number of enrolled and passed candidates from these five institutes:

From the above table, we can say that there are three institutes (A, B, and E) in which the number of
candidates passed is the same.
38. E
Sol. The table given below shows the number of enrolled and passed candidates from these five institutes:

Required percentage = = 16.04% ≈ 16%

Hence, option E is correct.


39. C
Sol. The table given below shows the number of enrolled and passed candidates from these five institutes:
www.byjusexamprep.com

Number of female candidates who passed in Institute C = = 144

Number of passed female candidates in Institute E = = 80

Number of passed female candidates in Institute F = = 112

Required average = = = 112

Hence, option C is correct.


40. C
Sol. The table given below shows the number of enrolled and passed candidates from these five institutes:

Number of candidates who passed in the exam of Institute M = 140 + 360 = 500

Number of enrolled candidates in Institute M = 2000 × = 2500

Success rate in Institute M = = 20%


www.byjusexamprep.com

Hence, option C is correct.


41. C
Sol.
Amount received by John from the ‘Profit’ scheme = P
John invested this amount in a scheme that offered 10% compound interest per annum for 1
year.

Amount received by John at the end of the year = P (1 + )


According to the question:

P (1 + ) = 19800

P (1.1) = 19800
P (1.1) = 13750
P = 12500
Thus, John invested $12500 in the ‘Profit’ scheme.
Required value = 2P = 25000
Hence, option C is the correct answer.

42. C
Sol. Let ; then,

or

So, the values of ‘y’ are


Here, only and are real roots.

Hence, option C is the correct answer.


www.byjusexamprep.com

43. B
Sol. Number of arithmetic questions = 12
Number of algebra questions = x
According to the question:

=
23 – 23x = 5 + 115x + 660
18 – 138x – 660 = 0
3 – 23x – 110 = 0
3 – 33x + 10x – 110 = 0
3x(x – 11) + 10(x – 11) = 0
(3x + 10)(x – 11) = 0
x cannot be negative, so x = 11
Hence, option B is the correct answer.

44. A
Sol. Let ‘X’ be the number of members in the club and ‘F’ be the fund required.

It is given that
So, …(1)
Let ‘A’ be the average amount raised from each dancer.

Hence, option A is the correct answer.


45. E
Sol.
www.byjusexamprep.com

=x

=x

=x

=x

x=
Hence, option E is the correct answer.

46. D
Sol. The table given below shows the details about the different branches of the multinational company (M):

From the above table, we find that the total number of post graduate employees is 9675.

Hence, option D is correct.


47. A
Sol. The table given below shows the details about the different branches of the multinational company (M):
www.byjusexamprep.com

In China and Japan, the average number of employees per branch is the same, i.e., 160. Hence, option A
is correct.
48. E
Sol. The table given below shows the details about the different branches of the multinational company (M):

Number of postgraduate male employees in China = 1800 (given)

Number of postgraduate female employees in China = 2688 – 1800 = 888

Required percentage = × 100 = 74%

Hence, option E is correct.


49. A
Sol. The table given below shows the details about the different branches of the multinational company (M):
www.byjusexamprep.com

USA:

India:

China:

Japan:

Russia:

Clearly, the required percentage is the second highest in the USA.


50. D
Sol.
Percentage of the amount received by Jessica = 100 = 40%
So, Jessica completed 40% of the work in 12 days.

Thus, Jessica will complete 100% of the work in 12 = 30 days.


Hence, option D is the correct answer.

51. E
Sol. Let the two numbers be 18a and 18b, where ‘a’ and ‘b’ are co-primes.
LCM of the two numbers = 18ab = 1386
www.byjusexamprep.com

ab = 77 = 1 77 = 7 11
So, the numbers can be 18, 1386 or 126, 198.
So, the sum of the two numbers can be 1404 or 324.
Hence, option E is the correct answer.

52. C
Sol. Let the total number of available units be 100x.

So, number of units of cloth which can be used as curtains = 50x

Number of units of cloth which can be used as bedsheets = 70x

Now, 100x = (50x – 120) + (70x – 120) + 120

20x = 120

So, 100x = 600

Hence, option C is the correct answer.

53. A
Sol. Let the selling price be Rs. x.

According to the question:

54. B
Sol. Let the roots of the equation be (n – 12), n, and (n + 12).
www.byjusexamprep.com

For any cubic equation:

The product of the roots is given by

Or

By trial and error, .

So, the roots are 3, 15, and 27.

Here, the sum of the product of the roots, taken two at a time, is given by .

or

Hence, option B is the correct answer.

55. E
Sol. Let the number of persons in the original group be ‘x’.

So, total work = 18 12x units …(i)


When 10 persons are excluded from the group, the remaining group can complete the same
work in 24 days, working for 15 hours a day.
So, total work = 24 15(x – 10) units …(ii)
Since the total work is same in both the cases, after equating (i) and (ii), we get the following:
18 12x = 24 15(x – 10)
3 x = 5(x – 10)
3x = 5x – 50
2x = 50
x = 25
www.byjusexamprep.com

Thus, total work = 18 12x = 5400 units


Number of workers in the new group = 25 + 5 = 30
Let the required number of days be ‘N’.
So, N 300 = 5400
N = 18
Thus, the work will be completed in 18 days.
Hence, option E is the correct answer.

56. D
Sol. Number of ways in which the driver of the SUV can be chosen = 4
Remaining seats = 6
Six persons can be arranged in these six seats in 6! ways.
Total number of ways = 4 6! = 2880
Hence, option D is the correct answer.

57. C
Sol. Given:


As m and n are greater than 1, the only way to write 247 as a product of two numbers is
.
Thus, and .
So, the values of m and n are 16 and 3, respectively.
Required product =
Hence, option C is the correct answer.
58. E
Sol.
www.byjusexamprep.com

It is given that CP( ) = MP( ).

=
Next scenario:

CP(1 x%) = MP(1 – 25%)(1 – )

(1 x%) = ( )( )
(1 x%) = 1
x% = 0%
Hence, option E is the correct answer.
59. A
Sol. There are two possibilities here.

Case I: Red apple from the 1st bag and green apple from the 2nd bag

Probability
Case II: Green apple from the 1st bag and red apple from the 2nd bag

Probability

So, the total required probability


Hence, option A is the correct answer.
60. E
Sol. Let the three-digit number be abc or 100a + 10b + c.
Number obtained by reversing its digits = cba = 100c + 10b + a
Statement I: The difference between the original number and the number obtained by
reversing its digit is 297.
So, 100a + 10b + c – 100c – 10b – a = 99a – 99c = 297
a–c=3
Thus, the values of (a, c) can be (3, 0), (4, 1), (5, 2), (6, 3), (7, 4), (8, 5) and (9, 6).
From the above data, we cannot find the unit digit of the three-digit number.
www.byjusexamprep.com

Thus, statement I alone is not sufficient to answer the question.


Statement II: The sum of the digits of the three-digit number is 14.
a + b + c = 14
From the above data, we cannot find the unit digit of the given number.
Thus, statement II alone is not sufficient to answer the question.

Statements I and II together:


The cases for the value of (a, c) are (3, 0),(4, 1), (5, 2), (6, 3), (7, 4), (8, 5), and (9, 6).
Sum of a, b, and c = 14
From the above data, there will be many possible values of the unit digit of the three-digit
number, i.e., 0, 1, 2, 3, 4, and 5.
So, we cannot find the unit digit of the three-digit number even by combining both the
statements.
Hence, option E is the correct answer.

61. D
Sol.
It is given that pipes X, Y, and Z together can fill a tank in 12 hours.
Statement I: Pipes X and Y together can fill the tank in 16 hours.

So, time taken by pipe Z alone to fill the tank = = 48 hours


From the above data, we cannot find the time taken by pipe Y alone to fill the tank.
Thus, statement I alone is not sufficient to answer the question.

Statement II: Pipes Y and Z together can fill the same tank in 18 hours.

So, time taken by pipe X alone to fill the tank = = 36 hours


From the above data, we cannot find the time taken by pipe Y alone to fill the tank.
So, statement II alone is not sufficient to answer the question.
www.byjusexamprep.com

Statements I and II together:


Time taken by pipe Z alone to fill the tank = 48 hours
Time taken by pipe X alone to fill the tank = 36 hours
Time taken by pipes X, Y, and Z together to fill the tank = 12 hours
So, we can find the time taken by pipe Y alone to fill the tank.
Thus, both the statements together are required to answer the question.
Hence, option D is the correct answer.

62. C
Sol.

Required ratio = 350 : 750 = 35 : 75 = 7 : 15


63. E
Sol.

Number of boys in the mechanical branch who want to pursue higher studies = 150 × = 105

Number of boys in the computer science branch who want to get jobs = 70 × = 60
www.byjusexamprep.com

Required difference = 105 – 60 = 45


64. C
Sol.

Students of the civil branch who want to do higher studies = 120 × = 74

Students of the civil branch who want to get jobs = 74 × 3 = 222

Total number of students in the civil branch = 222 + 74 = 296


65. D
Sol.

Number of girls in the computer science branch = = 120

Number of girls in the computer science branch who want to pursue higher studies = 120 × = 108

Number of boys in the computer science branch who want to pursue higher studies = 210 – 108 = 102

Number of girls in the computer science branch who want to do job = (120 – 108) = 12

Number of boys in the computer science branch who want to do job = 70 – 12 = 58

Required ratio = 58 : 102 = 29 : 51


www.byjusexamprep.com

66. D
Sol. The least possible three-digit number that leaves a remainder of 24 when divided by 26 is 102.
The largest possible three-digit number that leaves a remainder of 24 when divided by 26 is
986.
The above numbers will form an arithmetic progression, and the common difference will be 26.
Let there be ‘n’ such numbers.
So, 986 = 102 + (n – 1)26
884 = (n – 1)26
34 = n – 1
n = 35
Thus, there are 35 such three-digit numbers that leave a remainder of 24 when divided by 26
Hence, option D is the correct answer.

67. E
Sol. Consider a 100 L solution.
Initially, there is 25 L of salt and 75 L of water.
Now, this 25 L is 40% of the solution.

So, the total solution = L


So, the amount of water evaporated = 100 – 62.5 = 37.5 L

Percentage =
Hence, option E is the correct answer.

68. D
Sol. Let the amount spent by each girl be Rs. x.
So, amount spent by each boy = Rs. (x + 98)
Total amount spent by the girls = Rs. 3x
Total amount spent by the boys = Rs. 4(x + 98)
According to the question:
www.byjusexamprep.com

4(x + 98) 3x = 10176


+ 98x = 848
+ 98x – 848 = 0
+ 106x – 8x – 848 = 0
x(x + 106) – 8(x + 106) = 0
(x – 8)(x + 106) = 0
Since the amount spent cannot be negative, x = 8
Thus, amount spent by each girl = Rs. 8
Hence, option D is the correct answer.

69. A
Sol. Let x, y, and z be the numbers of students who ate exactly 1 flavour, 2 flavours, and 3 flavours,
respectively.
Given: , , or
Thus, or
Number of children who did not eat any flavours = 60 – (x + y + z) = 60 – (15 + 25 + 15) = 5
Hence, option A is the correct answer.
70. A
Sol. Using statement I: As it is divisible by 11, ; and to be a multiple of 8, the possible
values of a and b can be (9, 0) or (1, 8). But as a < b so (9, 0) is not possible. Hence, a = 1
and b = 8
So, this statement is sufficient.
Using statement II: As it is divisible by 4, the possible value of b = 0, 4, 8 Also, as it is divisible
by 3, the possible values of (a, b) = (3, 0), (6, 0), (9, 0), (2, 4), (5, 4), (8, 4), (1, 8), (4, 8), (7, 8).
But as a < b so possible values are (2, 4), (1, 8), (4, 8), and (7, 8)
So, this statement is insufficient.
Hence, option A is the correct answer.
www.byjusexamprep.com

71. E
Sol. Using statement I: (Insufficient)
Using statement II: (Insufficient)
Using both the statements: (Insufficient)
Hence, option E is the correct answer.
72. D
Sol.
Number of triangles that can be formed using ‘n’ points =
According to the question:

= 84
n(n – 1)(n – 2) = 504 = 9 8 7
Comparing both sides, we get:
n=9

Now, number of triangles that can be formed using 2n or 18 points = = 816


Hence, option D is the correct answer.

73. B
Sol.
Here, one alphabet gets skipped each time a pair of two alphabets is selected. The numbers are derived
based on the position of the alphabet in the English alphabet as follows:
E (5) = 52 – 5 = 20; 20 ÷ 2 = 10, E10

G (7) = 72 – 7 = 42; 42 ÷ 2 = 21, G21


I (9) = 92 – 9 = 72; 72 ÷ 2 = 36, I36
K (11) = 112 – 11 = 110; 110 ÷ 2 = 55, K55
M (13) = 132 – 13 = 156; 156 ÷ 2 = 78, M 78
Hence, option B is the correct answer.
www.byjusexamprep.com

74. B
Sol. As we move diagonally below, we can observe that the sequence follows the pattern of addition of
numbers 3 and 4 to the previous number alternatively, starting with the addition of 3 and this process is
repeated continuously.

As 3 + 3 = 6 and 6 + 4 = 10, 10 + 3 = 13, and so on.

So, the required number will be 27 + 4 = 31. Hence, option B is correct.


75. B
Sol.
The protest is against the proposal itself, not the fee. So, there is no point in increasing the
fees of the licence. Postponing the implementation of the proposal and inviting the views of the
public before implementing it is a suitable course of action.

Hence, the correct answer is option B.

76. E
Sol. Neither A nor B follows due to the following reasons:

I is incorrect. Since the children are frightened of the pandemic, arranging sports and fun
activities in crowded settings is not desirable.
II is incorrect as it involves threatening, which is extreme.

Hence, the correct answer is option E.

77. A
Sol.
Course of action III can be rejected as it suggests resuming the power supply immediately. It is
mentioned in the statement that the power supply has been stopped as a precautionary
measure. Resuming it without paying attention to the issues that it might create is inadvisable.
So, course of action III does not follow.

I and II will follow as both of them are suggesting measures to help the people of Chennai in a
safe and logical way.
www.byjusexamprep.com

Hence, the correct answer is option A.

78. B
Sol.
Since going to bed or taking rest straight after dinner is not recommended, I does not follow.

Course of action II suggests a reasonable method of making people aware of the practices
that they should and should not do. So, course of action II follows. Hence, the correct answer
is option B.

79. B
Sol. In each step, one word and one number are rearranged.

Words are arranged alphabetically according to the last letter of each word. The first word according to
this order is placed at the rightmost position.

The numbers are arranged in increasing order. First, all the odd numbers are arranged and then the even
numbers are arranged (both in increasing order). The first number according to this order is placed at the
leftmost position.

In the second step, the next word according to the order specified above is put at the rightmost position;
the previous word then shifts leftwards. Also, the next number according to the order specified above is
put at the leftmost position and the previous number then shifts rightwards.

Input: butter range 20 pillow sick 25 35 rich 45 40

Step I: 25 butter 20 pillow sick 35 rich 45 40 range

Step II: 35 25 butter 20 pillow sick 45 40 range rich

Step III: 45 35 25 butter 20 pillow 40 range rich sick

Step IV: 20 45 35 25 pillow 40 range rich sick butter

Step V: 40 20 45 35 25 range rich sick butter pillow

‘butter’ is at the rightmost position in Step IV. Hence, option B is correct.


www.byjusexamprep.com

80. D
Sol. In each step, one word and one number are rearranged.

Words are arranged alphabetically according to the last letter of each word. The first word according to
this order is placed at the rightmost position.

The numbers are arranged in increasing order. First, all the odd numbers are arranged and then the even
numbers are arranged (both in increasing order). The first number according to this order is placed at the
leftmost position.

In the second step, the next word according to the order specified above is put at the rightmost position;
the previous word then shifts leftwards. Also, the next number according to the order specified above is
put at the leftmost position and the previous number then shifts rightwards.

Input: butter range 20 pillow sick 25 35 rich 45 40

Step I: 25 butter 20 pillow sick 35 rich 45 40 range

Step II: 35 25 butter 20 pillow sick 45 40 range rich

Step III: 45 35 25 butter 20 pillow 40 range rich sick

Step IV: 20 45 35 25 pillow 40 range rich sick butter

Step V: 40 20 45 35 25 range rich sick butter pillow

Clearly, ‘40’ will be at the leftmost position in Step V. Hence, option D is correct.
81. E
Sol. In each step, one word and one number are rearranged.

Words are arranged alphabetically according to the last letter of each word. The first word according to
this order is placed at the rightmost position.

The numbers are arranged in increasing order. First, all the odd numbers are arranged and then the even
numbers are arranged (both in increasing order). The first number according to this order is placed at the
leftmost position.
www.byjusexamprep.com

In the second step, the next word according to the order specified above is put at the rightmost position;
the previous word then shifts leftwards. Also, the next number according to the order specified above is
put at the leftmost position and the previous number then shifts rightwards.

Input: butter range 20 pillow sick 25 35 rich 45 40

Step I: 25 butter 20 pillow sick 35 rich 45 40 range

Step II: 35 25 butter 20 pillow sick 45 40 range rich

Step III: 45 35 25 butter 20 pillow 40 range rich sick

Step IV: 20 45 35 25 pillow 40 range rich sick butter

Step V: 40 20 45 35 25 range rich sick butter pillow

‘pillow’ will be in sixth place from left in Step III. Hence, option E is correct.
82. A
Sol. In each step, one word and one number are rearranged.

Words are arranged alphabetically according to the last letter of each word. The first word according to
this order is placed at the rightmost position.

The numbers are arranged in increasing order. First, all the odd numbers are arranged and then the even
numbers are arranged (both in increasing order). The first number according to this order is placed at the
leftmost position.

In the second step, the next word according to the order specified above is put at the rightmost position;
the previous word then shifts leftwards. Also, the next number according to the order specified above is
put at the leftmost position and the previous number then shifts rightwards.

Input: butter range 20 pillow sick 25 35 rich 45 40

Step I: 25 butter 20 pillow sick 35 rich 45 40 range

Step II: 35 25 butter 20 pillow sick 45 40 range rich

Step III: 45 35 25 butter 20 pillow 40 range rich sick


www.byjusexamprep.com

Step IV: 20 45 35 25 pillow 40 range rich sick butter

Step V: 40 20 45 35 25 range rich sick butter pillow

45 35 25 butter 20 pillow 40 range rich sick

The above sequence of words and numbers are Step III according to the logic being followed. Hence,
option A is correct.
83. A
Sol.
Only argument I is strong as it takes the instances into consideration and implies that animals
should not be used in movies. It is the empirical and logical basis that makes the argument
strong. Argument II is not a logical argument as it is too broad in its meaning.

Hence, the correct answer is option A.

84. B
Sol.
Argument I is not strong as it is more of a judgement and an opinion than a logical remark.
Argument II is strong as it provides a logical explanation and takes into account every
country's uniqueness and its needs.

Hence, the correct answer is option B.

85. D
Sol.
I is not a strong argument. Just because the number of fires are miniscule does not mean that
fire safety systems should not be installed in high-rise buildings.

II is not a strong argument. Costs cannot take precedence over safety.

Hence, the correct answer is option D.

86. D
Sol. N is older than the one who likes Gogole.
www.byjusexamprep.com

N > Gogole …(i)

S is younger than P.
P > S …(ii)
Q and N are younger than the one who likes OG, who is younger than the one who likes Tara and S.
Tara, S > OG > Q, N …(iii)
P is older than the one who likes Tara, who in turn is older than the one who likes Gogole.
P > Tara > Gogole …(iv)
At least one person is younger than the one who likes Pony.

R is just younger than Q …(v)


After combining (ii), (iii), and (v), we will get the following possibilities:
I) P > S, Tara > OG > Q > R > N

II) P > S, Tara > OG > N > Q > R

The one who likes Pony is younger than Q, and at least one person is younger than the one who likes
Pony.
From the above points, it can be concluded that the one who likes Pony is either the second or third
youngest.
[There can be two possible cases here.]
Case 1:

Case 2:

The one who likes Zipro is just older than the one who likes Xord, who is younger than the one who likes
OG.
www.byjusexamprep.com

S is younger than P, but S’ age is not closest to that of P’s..


Case 1:

Case 2:

N does not like Gogole and Papple.


The one who likes Papple is neither the youngest nor the oldest.
Case 2 gets eliminated as N > Gogole and, here, as there is no possible place left for the person who likes
Papple.

There are as many persons who are younger than the R as those who are older than S.
R is just younger than Q.
L is older, but L’s age is not closest to that of M’s. M is older than Q.

The oldest person, therefore, likes the Tita brand, while the third youngest person likes the Xord brand.
Hence, option D is the correct answer.
87. C
www.byjusexamprep.com

Sol. N is older than the one who likes Gogole.


N > Gogole …(i)

S is younger than P.
P > S …(ii)
Q and N are younger than the one who likes OG, who is younger than the one who likes Tara and S.
Tara, S > OG > Q, N …(iii)
P is older than the one who likes Tara, who in turn is older than the one who likes Gogole.
P > Tara > Gogole …(iv)
At least one person is younger than the one who likes Pony.

R is just younger than Q …(v)


After combining (ii), (iii), and (v), we will get the following possibilities:
I) P > S, Tara > OG > Q > R > N

II) P > S, Tara > OG > N > Q > R

The one who likes Pony is younger than Q, and at least one person is younger than the one who likes
Pony.
From the above points, it can be concluded that the one who likes Pony is either the second or third
youngest.
[There can be two possible cases here.]
Case 1:

Case 2:

The one who likes Zipro is just older than the one who likes Xord, who is younger than the one who likes
OG.
www.byjusexamprep.com

S is younger than P, but S’ age is not closest to that of P’s..


Case 1:

Case 2:

N does not like Gogole and Papple.


The one who likes Papple is neither the youngest nor the oldest.
Case 2 gets eliminated as N > Gogole and, here, as there is no possible place left for the person who likes
Papple.

There are as many persons who are younger than the R as those who are older than S.
R is just younger than Q.
L is older, but L’s age is not closest to that of M’s. M is older than Q.

Therefore, two people are older than the one who likes Papple.
Hence, option C is the correct answer.
88. B
www.byjusexamprep.com

Sol. N is older than the one who likes Gogole.


N > Gogole …(i)

S is younger than P.
P > S …(ii)
Q and N are younger than the one who likes OG, who is younger than the one who likes Tara and S.
Tara, S > OG > Q, N …(iii)
P is older than the one who likes Tara, who in turn is older than the one who likes Gogole.
P > Tara > Gogole …(iv)
At least one person is younger than the one who likes Pony.

R is just younger than Q …(v)


After combining (ii), (iii), and (v), we will get the following possibilities:
I) P > S, Tara > OG > Q > R > N

II) P > S, Tara > OG > N > Q > R

The one who likes Pony is younger than Q, and at least one person is younger than the one who likes
Pony.
From the above points, it can be concluded that the one who likes Pony is either the second or third
youngest.
[There can be two possible cases here.]
Case 1:

Case 2:

The one who likes Zipro is just older than the one who likes Xord, who is younger than the one who likes
OG.
www.byjusexamprep.com

S is younger than P, but S’ age is not closest to that of P’s..


Case 1:

Case 2:

N does not like Gogole and Papple.


The one who likes Papple is neither the youngest nor the oldest.
Case 2 gets eliminated as N > Gogole and, here, as there is no possible place left for the person who likes
Papple.

There are as many persons who are younger than the R as those who are older than S.
R is just younger than Q.
L is older, but L’s age is not closest to that of M’s. M is older than Q.

Therefore, ‘M is older than R, and M’s age is closest to that of S’ is the only true statement among the
given statements.
Hence, option B is the correct answer.
www.byjusexamprep.com

89. B
Sol. N is older than the one who likes Gogole.
N > Gogole …(i)

S is younger than P.
P > S …(ii)
Q and N are younger than the one who likes OG, who is younger than the one who likes Tara and S.
Tara, S > OG > Q, N …(iii)
P is older than the one who likes Tara, who in turn is older than the one who likes Gogole.
P > Tara > Gogole …(iv)
At least one person is younger than the one who likes Pony.

R is just younger than Q …(v)


After combining (ii), (iii), and (v), we will get the following possibilities:
I) P > S, Tara > OG > Q > R > N

II) P > S, Tara > OG > N > Q > R

The one who likes Pony is younger than Q, and at least one person is younger than the one who likes
Pony.
From the above points, it can be concluded that the one who likes Pony is either the second or third
youngest.
[There can be two possible cases here.]
Case 1:

Case 2:

The one who likes Zipro is just older than the one who likes Xord, who is younger than the one who likes
www.byjusexamprep.com

OG.

S is younger than P, but S’ age is not closest to that of P’s..


Case 1:

Case 2:

N does not like Gogole and Papple.


The one who likes Papple is neither the youngest nor the oldest.
Case 2 gets eliminated as N > Gogole and, here, as there is no possible place left for the person who likes
Papple.

There are as many persons who are younger than the R as those who are older than S.
R is just younger than Q.
L is older, but L’s age is not closest to that of M’s. M is older than Q.

Therefore, the age of the one who likes Papple is closest and lower to that of the one who likes Tara.
Hence, option B is the correct answer.
90. D
www.byjusexamprep.com

Sol. We will take the extreme case here where markets = industries = sectors. According to this
case, everything overlaps and there is no scope of a negative statement. Hence, none of the
conclusions fol7low.

Hence, the correct answer is option D.

91. E
Sol. Rajma chawal is made after Tuesday.
S made the dish (which is not korma) after rajma chawal but not on the next day.

Two dishes are made in the period between which shahi paneer and the dish made by S are prepared.
R made the dish after Monday but did not make shahi paneer nor rajma chawal.
Lemon rice was made immediately before biryani, which was not made by S.
www.byjusexamprep.com

Exactly one dish is made in the period between which biryani and the one made by Q are prepared. So, Q
made rajma chawal.
U did not make biryani or shahi paneer. So, U made lemon rice.
V made the dish after Monday but did not make chole. So V made shahi paneer.

Therefore, this is the final arrangement.


Five dishes are made before chole.

Hence, option E is the correct answer.


92. C
Sol. Rajma chawal is made after Tuesday.
S made the dish (which is not korma) after rajma chawal but not on the next day.
www.byjusexamprep.com

Two dishes are made in the period between which shahi paneer and the dish made by S are prepared.
R made the dish after Monday but did not make shahi paneer nor rajma chawal.
Lemon rice was made immediately before biryani, which was not made by S.

Exactly one dish is made in the period between which biryani and the one made by Q are prepared. So, Q
made rajma chawal.
U did not make biryani or shahi paneer. So, U made lemon rice.
V made the dish after Monday but did not make chole. So V made shahi paneer.

Therefore, this is the final arrangement.


www.byjusexamprep.com

From the final arrangement, we find that T made biryani on the day immediately before Tuesday.

Hence, option C is the correct answer.


93. D
Sol. Rajma chawal is made after Tuesday.
S made the dish (which is not korma) after rajma chawal but not on the next day.

Two dishes are made in the period between which shahi paneer and the dish made by S are prepared.
R made the dish after Monday but did not make shahi paneer nor rajma chawal.
Lemon rice was made immediately before biryani, which was not made by S.

Exactly one dish is made in the period between which biryani and the one made by Q are prepared. So, Q
made rajma chawal.
U did not make biryani or shahi paneer. So, U made lemon rice.
V made the dish after Monday but did not make chole. So V made shahi paneer.
www.byjusexamprep.com

Therefore, this is the final arrangement.

‘V: S’ is the odd one out as the two terms are related to consecutive days in each of the other options, but
there is a gap of two days between the days on which V and S cooked their dishes.

Hence, option D is the correct answer.


94. C
Sol. Rajma chawal is made after Tuesday.
S made the dish (which is not korma) after rajma chawal but not on the next day.

Two dishes are made in the period between which shahi paneer and the dish made by S are prepared.
R made the dish after Monday but did not make shahi paneer nor rajma chawal.
Lemon rice was made immediately before biryani, which was not made by S.
www.byjusexamprep.com

Exactly one dish is made in the period between which biryani and the one made by Q are prepared. So, Q
made rajma chawal.
U did not make biryani or shahi paneer. So, U made lemon rice.
V made the dish after Monday but did not make chole. So V made shahi paneer.

Therefore, this is the final arrangement.

‘Lemon rice’ is made on Sunday.

Hence, option C is the correct answer..


95. C
Sol.
From the context, it is clear that the advantages of fruits clearly make them a healthy food
option that will lead one to a healthy lifestyle as well. Thus, option C is clearly the right answer.

Option A cannot be determined from the passage.


www.byjusexamprep.com

Option B brings in a new element—vegetables.

Options D and E have not been stated in the passage.

Hence, the correct answer is option C.

96. A
Sol.
Clearly, the inference would be option A. Since the Russia-Ukraine war has led to the rise in
the prices of fossil fuels, it is clear that the supply of fossil fuels must have been reduced.

Hence, the correct answer is option A.

97. C
Sol.
The context clearly suggests that any movie, irrespective of its genre, cast, and anything else
is bound to succeed if it strikes a chord with the audience. The aforementioned idea is best
captured in option C, and hence it is the right answer here.

Option E is incorrect because even if the content is solid, its relatability still matters. If a movie
is complex, it might not find the required connection with the audience as well.

Hence, the correct answer is option C.

98. C
Sol. Since the tomb has been gaining attraction on social media, it can definitely be inferred that its
unusual message is the reason behind it. Since any unusual activity or a thing is likely to
capture one’s attention, option C can be concluded.
Hence, the correct answer is option C.
www.byjusexamprep.com

99. C
Sol.
Since it has been clearly mentioned that Mumbai is sinking, it can be clearly inferred that its
coastal regions in Mumbai need to be protected from sinking into the sea.

Hence, the correct answer is option C.

100. A
Sol.
G sits at fourth place to the left of E.

There are two people between E and F.


There will be 2 cases here.
www.byjusexamprep.com

One person sits between F and I.


I and G are not neighbours.
E and I are not neighbours.
This eliminates Case 2 as, here, I will have to be a neighbour of either E or G.

G sits to the immediate left of J.


www.byjusexamprep.com

J sits fourth to the right of K.


Two people sit between K and I.

This is the final arrangement.

Therefore, 15 people were sitting around the table before 5 new people joined in. So, the total number of
people sitting around the table is (15 + 5) = 20
Hence, option A is the correct answer.
101. C
Sol.
Since,
www.byjusexamprep.com

The total number of students who are good at mathematics as well as sociology is (5 + 2) = 7.

But 2 students out of these are good in accountancy.

So, the number of students who are good in both mathematics and sociology but not good in accountancy
is 5.
102. A
Sol.
The possible venn diagram for the given statements is as follows:

I. Some ink can be polymer → It’s possible as there is no relation mentioned between ink and polymer.
II. All ink are not clay → It’s possible but not definite as there is no relation between ink and clay.
Therefore, only I follows.
Hence, option A is the correct answer.
103. B
Sol. Since a big goal was announced by the government to reduce the level of water pollution in
Yamuna River by processing close to 95 percent of wastewater, the intentions of the
government are clear—that it is adamant on reducing pollution levels in Yamuna River and
Delhi. The given sentiment is best captured in option B, and hence it is the right answer here.
Hence, the correct answer is option B.

104. C
Sol. Except option C, the rest of the answer options can either be inferred from the given context or are true in
www.byjusexamprep.com

the given context. Option C takes a different route by stating that Hydrogen can fulfil the needs of the
countries across the world due to its abundance. Since this is something that is taken for granted, it is an
assumption.
Hence, the correct answer is option C.

105. D
Sol. Except option D, here also, the rest of the answer options can be inferred or derived from the
paragraph. Option D talks about people’s messed up genes making them more likely to suffer
from mental illness. However, as far as the context goes, we can’t compare the given two
factors that can lead to mental illness. Hence, it can only be termed as an assumption.
Hence, the correct answer is option D.

106. A
Sol. In order to prove that the games are not becoming redundant, we need to make sure we pick
an answer option that talks about Commonwealth games receiving significantly great
response as opposed to what was expected on them. Out of the given options, only option A
manages to achieve the same. Hence, it is the right answer here. Option E sounds more like
an opinion than an argument, and it can’t be true for each and every person. So, it is rejected.
Hence, the correct answer is option A.

107. B
Sol. Here, we’ll need an answer option that provides support to farm employment.
Out of the given options, only options B and D provide support for the given cause. And even
option D is more of a historical precedent than an argument in favour of farm employment
because we can’t predict future events solely based on past performance. So, option B is the
only logical explanation here. Option E is incorrect because even though farm employment
has provided sustenance to people since ages, it does not mean that this should continue.
Hence, the correct answer is option B.
108. D
Sol. Working from top to bottom, the right-hand box of each row of the table has double the number of the
www.byjusexamprep.com

number present in the right-hand box of the previous row of the table. 2 is written in the right side of the
first row and the numbers go on doubling the previous value continuously.

The difference between the larger and the smaller numbers in each row is put in the left-hand box on the
row below.

So, 32 – 9 = 23 is the required number. Hence, option D is correct.

You might also like